Sei sulla pagina 1di 356

All Pediatric Seminars

Objectives
This work was done by :
Only and only use the objectives , mostly from the references : USMLE Step 2
CK l Pediatrics + Illustrated Textbook of Paediatrics + Nelson Essentials of
Pediatrics 7th Edition
Last update at :
Topics :

 1- Approach to a child with skin rash


 2- Approach to a febrile infant
 3- Approach to syncope
 4- Approach to a child with heart failure
 5- Approach to a cyanotic infant
 6- Approach to failure to thrive
 7- Approach to proteinuria
 8- Approach to Hematuria and UTI
Topics :

 9 - Child with chronic diarrhea


 10- Child with Developmental Delay & Cerebral palsy
 11- Child with Rickets
 12- Childhood obesity
 13- Approach to a wheezy infants
 14- Allergies in Pediatrics
 15- Approach to child with Hemolytic Anemia (Pallor and
Jaundice)
 16 -Approach to a child with abdominal pain
Topics :

 17- Approach to suspected child Abuse


 18 – Approach to unconjugated Hyperbilirubinemia
 19- Approach to conjugated hyperbilirubinemia
 20- Child with recurrent infection (immunodeficiency & HIV
Infection)
 21 -Approach to lymphadenopathy
 22- Approach to a limping child
 23- Child with bleeding disorder
 24- Approach to abnormal head size
Approach to child with
rash

References for this topic : nelson+ Illustrated Textbook of Paediatrics


Objectives

• Describe the different morphological types of rash.


• List and discuss the common infectious causes of the following:
1. Vesicular rash
2. Macular and popular skin rash
3. Maculopapular rash
4. Peticheal rash
5. Purpuratic rash
• Recognize the key component from the history and physical
examination of a rash.
• Outline the initial evaluation for skin rashes
Morphologically

A) Primary: is defined as the basic lesion


that arises at the beginning and is most
characteristic of the disease process.

B) Secondary: are the residue, or result, of


the effects of the primary lesion. They may be
created by scratching or secondary infection.
 History:
 The age of the patient, onset, duration, progression, associated cutaneous
symptoms (pain, pruritus), and associated systemic signs or symptoms
(fever, malaise, weight loss) are important clues.

 Other important information includes a history of allergies, environmental


exposure, travel history, previous treatment, affected contacts, and family
history.
 Examinaton:

 A careful examination of the skin requires a visual


and a tactile assessment.

 Examination of the skin over the entire body must


be performed systematically.

 Mucous membranes, hair, and nails, all of


ectodermal origin, also may be involved in
cutaneous disorders and should be assessed.
Infectious Causes of
Skin Rash

Vesicular
Rash

Petechial
Macular and
and
Maculopapul
Purpuric
ar Rash
Rash
Vesicular Rash

Viral Bacterial

Herpes Varicella- Coxsackie Staphylococcu


Group A
Simplex Virus Zoster Virus virus s
Streptococus

Chickenpox shingles
hand-foot-mouth
Syndrome
Varicella “‫” العنقز‬ Multiple stages of the rashes
-Cause by  Varicella Zoster virus , herpes virus
-Clinically : fever, headache , malaise
macules , papules , Vesicles and crusts in Various stages of healing
-Management : Supporative in immunocompetent
Consider Acyclovir , VZIG
-Complications :
Pneumonia
Encephalitis
Congenital varicella
Skin infection

-Management : Supporative
-Complication: Viral Myocarditis
Macular and Maculopapular Rash

Viral Bacterial

Group A
Adenovirus HHV6,7 Parvovirus Salmonella
Streptococcus
Roseola infantum(sixth disease)
-Cause by HSV-6
-Clinically : high fever (40)
After the fever Subsides Pink-colored maculopapular rash appear
-Management : Supportive
-Complications: “ Febrile seizure “

8. Scarlett fever ‫ىاقرمزية‬


‫اال‬
‫االاحم‬
-Cause by  group A beta-hemolytic streptococci (GABHS)
-Clinically : Exudative pharyngitis (sore throat) + strawberry tongue
Fine maculopapular rash “Sand paper” + Pastia lines
-Management : Penicillin
-Complications : Acute Rhumatic Fever and acute poststeptococcal glumerlonephritis(APSGN)
Petechial and Purpuric Rash

Viral Bacterial

Adenovirus Enterovirus Rubella Meningococcal HIB


 Meningococcal Septicemia :
 Life threatening condition
 Lethargy, fever accompanied by purpuric rash
 Caused by meningococcal infection
 Systemic antibiotic immediately “penicillin”
 Admission to hospital
Approach to a febrile
infant

References for this topic : USMLE Step 2 CK l Pediatrics + Illustrated Textbook of


Paediatrics + Nelson Essentials of Pediatrics 7th Edition
Objectives :

 Define fever and its patterns


 Define FWLF (Fever without localizing focus)
 Discuss the approach to a well febrile child FWLF
 Discuss the approach to a sick febrile child with FWLF
 List the differential diagnosis of fever with rash.
 Describe different types of rash.
 Discuss clinical features of common pediatric exanthema.
 Discuss the investigation of common pediatric exanthema.
 Define PUO.
 List possible cause .
 Discuss the approach to PUO.
 Discuss the management.
Define fever and its patterns
 Definition : Fever, also known as pyrexia and febrile : is a normal body
response to a variety of conditions, the most common being infection .
  Core body temperature is normally maintained within 1C to 1.5 C in a range of
37 C to 38 C.

 The pattern : of fever in children depending on age and the nature of the
illness:
1. Neonates may not have fever but be hypothermic, despite significant infection.
2. older infants and children younger than 5 years of age may have an
exaggerated febrile response with temperatures of up to 105° F (40.6° C) in
response to either a serious bacterial infection or an otherwise benign viral
infection.
3. in older children and adolescents Fever up to 105° F (40.6° C) suggests a
serious process.
How is fever identified in children? In hospital, it is measured at:
• <4 weeks old by an electronic thermometer in the axilla
• 4 weeks to 5 years by an electronic or chemical dot thermometer in
the axilla or infrared tympanic thermometer.
 In general, axillary temperatures underestimate body temperature
by 0.5°C.
Define FWLF (Fever without
localizing focus)
 Most febrile illnesses in children may be
categorized as follows:
 • Fever of short duration accompanied by localizing signs and
symptoms, in which a diagnosis can often be established by clinical
history and physical examination .
 • Fever without localizing signs (fever without a focus) ,
frequently occurring in children younger than 3 years of age, in which
a history and physical examination fail to establish a cause.
 • Fever of unknown origin (FUO) , defined as fever for >14 days
without an identified etiology despite history, physical examination,
and routine laboratory tests or after 1 week of hospitalization and
evaluation.

Chapter 96 u Fever Without a Focus  3


Discuss the approach to a well febrile child FWLF

 FEVER IN INFANTS YOUNGER THAN 3 MONTHS OF AGE : causes ?


•    bacteremia : in neonates : bacteremia (caused by group B streptococcus [GBS],
Escherichia coli, and Listeria monocytogenes .in 1- to 3-month-old infants : Streptococcus
pneumoniae, Haemophilus influenzae, non typhoidal Salmonella, and Neisseria
meningitidis .
•  urinary tract infection (UTI) :  (E. coli)
• pneumonia  (S. pneumoniae, GBS, or  Staphylococcus aureus).
• meningitis (S.  pneumoniae, H. influenzae type b, GBS,  N. meningitidis, herpes simplex
virus [HSV], enteroviruses).
• bacterial diarrhea  (Salmonella, Shigella, E. coli).
• osteomyelitis or septic arthritis (S. aureus or GBS).
 FEVER IN CHILDREN 3 MONTHS TO 3 YEARS OF AGE :
•  most have self-limited viral infections, some have occult bacteremia (bacteremia
without identifiable focus) or UTIs, and a few have severe illnesses.
•  Observational assessment is a key :  alertness ,  irritability ,  sitting, moving
arms and legs ,  playing with objects .
Discuss the approach to a sick febrile child with FWLF
 Admitted to the hospital ? Febrile
infants <3 months of age who appear ill
or all febrile Infants <4 weeks of age
should be for empirical antibiotics
pending culture results.
 After blood, urine, and cerebrospinal fluid
cultures are obtained, broad-spectrum
parenteral antibiotics (typically
ampicillin with cefotaxime or gentamicin)
are administered.
 What is the septic screen ? Full blood
count including differential white cell
count . Blood culture . Urine for
microscopy ,culture and sentivity . Acute
phase reactant e.g. C-reactive protein .
CSF (unless contraindicated ) for
microscopy , culture and sensitivity . Figure 96-1 Approach to a child younger than
chest X-ray . 36
months of age with fever without localizing
  Well-appearing febrile infants ≥ 4 weeks signs. The
of age without an identifiable focus and specific management varies, depending on the
age and
with certainty of follow-up are at a low clinical status of the child.
risk of developing a serious bacterial
List the differential
diagnosis of fever
with rash
251-Infection and immunity -
Illustrated Textbook of Paediatrics
Chapter 97 u Infections Characterized by Fever and Rash  329
Describe different types of rash

Macule Papule Maculopapular

A ‘macule‘ is a small, A ‘papule‘ is a small, The term ‘maculopapular‘


flat area (or ‘spot’) of raised area of skin (less refers to a combination of
discoloured skin (usually than 5 mm). It usually small flat ‘spots’ and raised
less than 5 mm in
has a domed top ‘spots’ on the skin. This
diameter).
(although it can be flat). sort of rash is typical of
measles.
Describe different types of rash

Vesicle Pustule Nodule

A ‘pustule‘ is a vesicle
A ‘vesicle‘ is a papule containing yellow fluid. This A ‘nodule‘ is a larger
with a fluid-filled centre. fluid usually consists of serum swelling on the skin surface
(blood fluid), white blood (usually more than 5 mm in
Vesicles are typical of cells, and the virus that has diameter). It extends deep
chickenpox rash and caused the original infection. into skin, and is usually firm
‘cold sores’. The presence of a pustule to the touch.
does NOT mean that the rash
has ‘become infected’ with
bacteria; pustules are an
‘expected’ event in many
viral illnesses.
Describe different types of rash
Purpura Petechiae

• Petechiae are small, red-brown, flat macules up to 2 mm


‘Purpura‘ means areas in diameter.
of little petechiae joined • It is important to note that they do not blanch (turn white)
together. These are when pressure is applied with a finger — in contrast to
therefore larger areas most rashes in children which DO fade when pressure is
(usually more than 2 applied.
mm) of bleeding under • Petechiae are caused by tiny spots of blood gathered under
the surface of the skin.
the skin.
• They are important in diagnosing meningococcal disease
and other conditions. Anyone with a non-blanching rash
needs to see a doctor promptly.
Discuss clinical features of common pediatric exanthema. AND
Discuss the investigation of common pediatric exanthema.

 An exanthem or exanthema (from Greek, "a breaking out”) widespread rash .


 Infectious exanthem : see the ospe summary or the common infection
summary .
1. (rubeola) measles "first disease”
2. rubella, ("German measles") "third disease”
3. erythema infectiosum "fifth disease”
4. roseola infantum "sixth disease”
5. Scarlet fever, or "second disease"
6. Varicella zoster virus (chickenpox or shingles)
7. Mumps
8. rhinovirus (the common cold)
Define PUO. List possible cause . Discuss the
approach to PUO. Discuss the management.

 Fever of unknown origin (FUO), pyrexia of unknown origin (PUO)


  FUO is defined as :
1. temperature greater than 100.4 F (38 C)
2. lasting for >14 days
3. without an obvious cause despite a complete history, physical
examination, and routine screening laboratory evaluation.

  Infections are the most common cause of FUO in children, followed by


inflammatory diseases, malignancy, and other etiologies .
 Unless the patient is acutely ill, no therapy should be started before the
cause has been found. This is because non-specific therapy is rarely
effective
Case#1
An 18-month-old male has an acute onset of fever Case#2
A seven-month-old girl presents with a fever to 39.5C,
to 39C in the pediatric emergency room. His
mild irritability, and poor feeding. How would you
history and physical exam do not reveal an proceed?
obvious source of infection. He looks generally
well (not "sick").
What percentage of children would have a
positive blood culture? Case#3
A six month old has had a high fever for three days and a
maculopapular rash. . What illness the child might have?
a. What organisms would you expect if the
How would you manage him?
blood culture were positive?
b. Why is bacteraemia dangerous?
c. Would you get a urine culture?
d. How would you obtain a urine culture (bag
Case#4
mid-stream or catheter or super pubic)?
A6 years old child present with history of fever for one
e. What percentage of urine cultures would be month. He received many courses of antibiotics with no
positive? improvement. He was found to have large spleen.
f. List three of the most likely viral infections How can you help him?
that could be causing the fever and what
findings would you look for?
Syncope

References for this topic : nelson+ Illustrated Textbook of Paediatrics


Define and discuss briefly the pathophysiology
behind syncope

 Syncope is the transient loss of consciousness and muscle tone .


 Syncope is relatively common .
 Most syncopal events are relatively benign (usually does not represent
cardiac disease) .
 but can represent a serious cardiac condition that may lead to death .
Recognize the features of syncope and how to
differentiate it from other causes of loss of
consciousness.

 differential diagnosis for typical syncope :


 includes seizure, metabolic cause
(hypoglycemia), hyperventilation,
atypical migraine, and breath holding .

 Typical syncopal events usually occur in the


upright position or are related to changing position .
 Syncope may be associated with anxiety, pain, blood
drawing or the sight of blood, fasting, a hot
environment, or crowded places .
 The patient often appears pale
 A prodrome, consisting of dizziness, lightheadedness
nausea, diaphoresis, visual changes (blacking out), or possibly
palpitations.
 Unconsciousness lasts for less than 1 minute .
 A return to normal consciousness occurs relatively quickly .
 Most of these syncopal episodes are vasovagal or
neurocardiogenic in origin.
 The physical examination is normal.
Discuss the most common causes of syncope in
childhood
 The causes are:
 Neurocardiogenic – prolonged standing and vagal symptoms .
 Situational – defecation, urination, cough or swallowing .
 Orthostatic – BP fall > 20 mmHg after 3 min .
 Ischemic .
 Arrhythmic – heart block, supraventricular tachycardia, ventricular tachycardia.

 In most, the cause is non-cardiac. Features suggestive


of a cardiac cause are:
• Symptoms on exercise – potentially dangerous
• Family history of sudden unexplained death
• Palpitations.
Identify the life threatening causes of syncope in
children
 Most commonly the cardiac
 serious cardiac condition that may lead to death
Explain the diagnostic investigation that is used in
evaluating syncope

 Syncopal episodes may require no more than reassurance to


the patient and family .
 If the episodes have a significant impact on daily activities further
evaluation may be indicated
 (ECG)

Check blood pressure and signs of cardiac disease (murmur, femoral pulses,
Marfan syndrome). ( this is clinical not diagnostic study but just for knowledge
).
Understand the short and long term risk associated
with syncope.

 Recurrent syncope has serious effects on quality of life. The impairment due to
syncope is comparable with chronic illnesses such as chronic arthritis, recurrent
moderate depressive disorders and end-stage kidney disease.
 Morbidity is particularly high in the elderly and includes loss of confidence,
reduced mobility, depressive illness, fear of falling, fractures and subsequent
institutionalisation.
 Female gender, a high level of comorbidity, the number of episodes of syncope
and the presence of presyncope seem to be associated with poorer quality of life.
 Physical injury: soft tissue and bone injuries may occur. Syncope was found to be
the cause of 21% of road accidents involving loss of consciousness at the wheel,
second only to epilepsy
 https://patient.info/doctor/syncope#nav-6
Heart failure

References for this topic : nelson+ Illustrated Textbook of Paediatrics


Objectives
 Define heart failure.
 List the risk factors.
 Discuss the clinical presentation.
 Discuss the investigation.
 Outline the management.
Definition
 Heart failure occurs when the heart is unable to pump blood at a rate
commensurate with metabolic needs (oxygen delivery).
Clinical presentation

 Heart failure presents in infants as poor feeding, failure to thrive,


tachypnea, and diaphoresis with feeding.
 Older children may present with shortness of breath, easy fatigability, and
edema.
 The physical examination findings depend on whether pulmonary venous
congestion, systemic venous congestion, or both are present.
 Tachycardia, a gallop rhythm, and thready pulses may be present with
either cause.
 If left-sidedfailure is predominant, tachypnea, orthopnea, wheezing, and
pulmonary edema are seen.
 Hepatomegaly, edema, and distended neck veins are signs of right-sided
failure.
Investigation.
 Investigations (heart failure is clinical diagnosis) The 4 major
features of HF are: Tachycardia, Tachypnea, Cardiomegaly, &
Hepatomegaly.
 1- Chest X-ray Cardiomegaly-plethoric lung (pulm vascular
marking)
 2- Echo Confirm left ventricle dysfunction (decreased ejection
fraction normal 55-65% & increased ejection time). Confirm
chamber enlargement. May detect cause of failure.
 3- ECG: Detect arrythmia.
 4- BNP (serum B-type Natriuretic Peptide), cardiac neurohormone
may ↑
TREATMENT
 Initial treatment is directed at improving myocardial function
and optimizing preload and afterload.
 Diuretics, inotropic support, and, often, afterload reduction are
employed (Table 145-3).
 Long-term therapy usually consists of diuretics and afterload
reduction is added.
 Long-term therapy with β-blockers also may be beneficial,
although this remains somewhat controversial in pediatric
patients.
 Spironolactone is usually added to the medical regimen
because of its effect on cardiac remodeling.
Approach to a
cyanotic infant
By : Asem M. Shadid – 435032467

References for this topic : USMLE Step 2 CK l Pediatrics + Illustrated Textbook of


Paediatrics + Nelson Essentials of Pediatrics 7th Edition
Objectives :

 1. Define cyanosis.
 2. Classify and enumerate the various causes of cyanosis.
 3. Outline methods to differentiate cardiac for non-cardiac causes
of cyanosis.
 4. Formulate an appropriate approach to evaluate an infant with
cyanosis.
Define cyanosis
  Acrocyanosis or Peripheral cyanosis (blue color of the hands
and feet with pink color of the rest of the body) is common in the
delivery room and is usually normal.  may occur when a child is
cold .
 Central cyanosis of the trunk, mucosal membranes, and tongue
can occur at any time after birth and is always a manifestation of a
serious underlying condition. cardiopulmonary diseases are the most
common causes .
  most common causes of cyanosis in infants admitted to a neonatal
intensive care unit : Respiratory distress  syndrome, sepsis, and
cyanotic heart disease .
 Differential cyanosis Upper half of the body is pink and the lower
half cyanotic, or vice versa .

 Cyanosis is noted with 4 to 5 g/dL of deoxygenated hemoglobin.


Bluish discoloration of the tissues results when the absolute level of
reduced hemoglobin in the capillary bed exceeds 4 g/dL .
causes of cyanosis :  presentation of congenital heart disease is with a
heart murmur, heart failure, cyanosis or shock
depends on the underlying anatomic lesion
 Cyanosis in a newborn infant with causing:
respiratory distress (respiratory
rate >60 breaths/min) may be • left to right shunt
due to:
• right to left shunt
• Cardiac disorders – cyanotic congenital
• common mixing
heart disease
• outflow obstruction in the well or sick child.
• Respiratory disorders, e.g. surfactant
deficiency, meconium aspiration,
Common Cyanotic Heart Disease
pulmonary hypoplasia, etc.
(5 Ts) :
• Persistent pulmonary hypertension of 1. Tetralogy of Fallot
the newborn (PPHN) – failure of the 2. Transposition of great vessels
pulmonary vascular resistance to fall 3. Truncus arteriosis
4. Total anomalous pulmonary
after birth venous return
• Infection – septicaemia from group B 5. Tricuspid atresia
streptococcus and other organisms
• Metabolic disease – metabolic acidosis
and shock.
differentiate cardiac for non-cardiac
causes of cyanosis
 breath sounds are usually normal in cardiac disease
 while wheezes, rhonchi, and chest wall abnormalities usually accompany a
pulmonary process.
 hyperoxia test :
• If a low-pulse oximeter reading persists, it may be appropriate to
proceed to a hyperoxia test. It is indicated if the pulse oximeter
reading is less than 85% in both room air and 100% oxygen .
• Useful in distinguishing cardiac from pulmonary causes of cyanosis.
1. cyanotic cardiac disease : little response to increased ambient oxygen
2. pulmonary disease : the saturation increase may be dramatic with Oxygen.
 ABG : elevated pCO2 usually not seen with cyanotic congenital heart disease
unless there is associated pulmonary congestion.
 Heart murmer : does not rule out cyanotic cardiac disease if not ; in most
conditions with right-to-left shunting there is no murmur.
Formulate an appropriate approach to evaluate an
infant with cyanosis.

 https://
www.slideshare.net/sunilagrawal9693/approach-to-a-neonate-with-cyano
sis
Approach to failure to thrive

References for this topic : USMLE Step 2 CK l Pediatrics + Illustrated Textbook of


Paediatrics + nelson
Define FTT
 child fails to gain weight appropriately or, in more severe
cases, experiences failure in linear growth or head
circumference.
 The term ‘failure to thrive’ is used to describe suboptimal,
weight gain in infants and toddlers .
Describe the normal
growth pattern in
full term & pre-term
infants
 Growth in the first year depend on nutrition (When caloric intake is
inadequate, the weight percentile falls first, then the height )
 childhood to the puberty depend on ( GH – THYROID )
 length decreases first or at the same time as weight (GH deficiency).
 Puberty – adolescence ( sex hormones )
 An infant typically doubles BW by 6 months and triples by 1 year.
 Between 10 and 20 years: acceleration in early adolescence. Boys’
highest growth stops at age 18.
 Their average peak is 13.5 years (2–3 years later than girls, and
continues 2–3years after girls have stopped). Girls’ average peak
is 11.5 years and it stops at age 16.
Plot growth parameters on growth charts

 ….
List the important points in the history (risk factors)

 A medical history should include prenatal history prematurity .


 birth size (weight, length, and head circumference), as well as
family and travel history .
 Indicators of medical diseases, such as vomiting, diarrhea,
fever, respiratory symptoms, and fatigue, should be noted .
 A careful diet history is essential .
 Lactation problems in breastfed infants and improper formula
preparation are frequent causes of growth failure early in
infancy .
 It is crucial to evaluate intake of solid foods and liquids for older
infants and children. Due to parental dietary beliefs .
 Some children have inappropriately restricted diets; others drink excessive amounts
 The child’s daily meal schedule (timing, frequency, location) .
 A complete psychosocial assessment of the child and family is
required.
 Child factors (temperament, development),
 parental factors (dépression, Domestic violence, social isolation,
mental retardation, substance abuse) .
 environmental and societal factors (poverty, unemployment,
illiteracy, lead toxicity) all may contribute to growth failure.
List the important physical signs on examination

 oral or dental problems .


 indicators of pulmonary, cardiac, or gastrointestinal or dental
problems disease
children with FTT have more otitis media, respiratory, and
gastrointestinal infections
 dysmorphic features that may suggest a genetic or
teratogenic cause for growth failure .
 A complete neurologic examination may reveal spasticity or
hypotonia, which can
have untoward effects on feeding and growth .
 Physical findings related to malnutrition include :
1. decreased subcutaneous fat .
2. decreased muscle mass .
3. dermatitis.
4. hepatomegaly .
5. Cheilosis .
6. edema

Cheilosis
Recognize the difference between
marasmus and dehydration
 Marasmus results from the
body’s physiologic response to
inadequate calories and
nutrients.
 Loss of muscle mass and
subcutaneous fat stores can be
confirmed by inspection or
palpation .
Dehydration

 Dehydration, most
often due to
gastroenteritis is
common in children
Discuss the classification of FTT
 Organic failure to thrive .

 Non-organic failure to thrive .


Describe non-organic FTT
 Child (usually infant) not fed adequate calories .
 Emotional or maternal deprivation concurrent with nutritional deprivation
.
 Leads to neglect of infant; psychosocial deprivation most common
reason in all age groups
 Other factors: retarded or emotionally disturbed parents; poverty .
 Clinical findings :
 Thin extremities, narrow face, prominent ribs, wasted buttocks
 Neglect of hygiene
 A flat occiput and hair loss may indicate excessive back-lying
 Delays in social and speech development
 –Avoidance of eye contact, expressionless, no cuddling response
 Feeding aversions
List the causes of organic FTT

 Malabsorption : infection, celiac disease, cystic


fibrosis, disaccharide deficiency, protein-losing
enteropathy .
 Allergies
 Immunodeficiency
 Chronic disease
List the differential
diagnosis
List the lab tests
 Initial diagnostic tests (when organic causes are suspected)
 document caloric intake.
 CBC, urinalysis, liver function tests, serum protein .
 sweat chloride .
 stool for ova and parasites
To increases your knowledge *
List the imaging studies

 Review the previous slide


Describe the dietary managements
 require intensive and long-term intervention
 usually require greater calories for catch-up growth. May need NG feedings
or even gastrostomy tube in severe cases .
 Nutritional rehabilitation should be initiated and advanced slowly to
minimize complication
 The initial approach involves correction of dehydration and anti-infective
(bacteria, parasites) therapy if indicated .
 Oral rehydration is recommended over intravenous fluid to avoid
excessive fluid
and solute load and resultant heart or renal failure.
 Remember : Nutritional rehabilitation can be complicated by refeeding
syndrome
Describe the monitoring indices

 ???
 Case#1  
A 6 year old boy has been seen for
preschool clinical assessment. Although
previously at the 25th percentile for
height and weight, he is now at the 5th
percentile for weight and has remained
at the 25th percentile for height. How
would you counsel him and his family?
 
 Case#2
An 8-month old has a weight, which is
below the fifth percentile. List five
organic causes of failure to thrive and
diagnostic hints.
Approach to
proteinuria

References for this topic : nelson+ Illustrated Textbook of Paediatrics


Objectives
 Define and classify proteinuria.
 Discuss the pathophysiology behind normal and abnormal protein
excretion.
 Discuss the various etiologies of proteinuria.
 Describe the different methods of measurement and detection of urinary
proteins.
 List the key component from history and physical examination of a child
with proteinuria.
 Explore the investigation and initial management plan for a patient with
proteinuria.
 Define nephrotic syndrome
 Recognize the clinical presentation
 Discuss the causes
 List the complications.
Definition
 Normally in healthy children small amounts of protein are found
<4 mg/m2/hour OR Urine Pr/Cr <0.2

 Proteinuria is defined as protein > 4mg/m2/hour OR Urine Pr/Cr >


0.2
Types
 Transient proteinuria may occur during febrile illnesses or
after exercise and does not require investigation.
Persistent proteinuria is significant and should be
quantified by measuring the urine protein/creatinine ratio
in an early morning sample (protein should not exceed 20
mg/mmol of creatinine).
 Orthostatic (postural) proteinuria, where proteinuria is
only found when the child is upright, i.e. during the day. It
can be diagnosed by measuring the urine
protein/creatinine ratio in a series of early morning urine
specimens. The prognosis is excellent and further
investigations are not necessary.
Persistent proteinuria :

 Tubular proteinuria it is characterized by increased filtration,


impaired reabsorption, secretion of low molecular weight proteins
in urine. It is suspected with : acute tubular necrosis, pyelonephritis,
polycystic kidney disease, tubular toxins ( EX : antibiotics and
chemotherapeutic agents )

 Glomerular proteinuria there are disruptions of normal glomerular


barrier to protein filtration leading to passage of BOTH large and
small molecular weight proteins in urine. Often there is glomerular
disease.
PATHOPHYSIOLOGY :

 Normal protein excretion  — Urinary protein excretion in


the normal child is less than 100 mg/m2 per day or a total of
150 mg per day. In neonates, normal urinary protein excretion
is higher, up to 300 mg/m2, because of reduced reabsorption
of filtered proteins.

 Abnormal protein excretion  Urinary protein


excretion in excess of 100mg/m2 per day or
4 mg/m2 per hour is considered abnormal in
children. Nephrotic range proteinuria (heavy
proteinuria) is defined as ≥1000 mg/m2 per day or
40 mg/m2 per hour.
MEASUREMENT OF URINARY PROTEIN

Qualitative Quantitative

• Urine dipstick • timed 24-hour


• Sulfosalicylic acid urine collection
test • measurement of
the urinary
protein/creatinine
ratio
MEASUREMENT OF URINARY
PROTEIN
Dipstick:
 Primarily detects albumin excretion and provide semi-quantitative estimate of
urinary protein.
 Protein reagents have 6 levels:

Negative <5 mg/dL


Trace 5-20 mg/dL
1+ 30 mg/dL
2+ 100 mg/dL
3+ 300 mg/dL
4+ >2000 mg/dL
MEASUREMENT OF URINARY PROTEIN

Quantitative assessment
 most common method - 24-hour urine collection
 Normal protein excretion
 Child: < 100mg/m2/day or 150mg/day
 Neonates: up to 300mg/m2/day
 In children: levels >100 mg/m2 per day (or 4 mg/m2 per hour)
are abnormal
 Proteinuria of greater than 40 mg/m2 per hour is considered
heavy or in the nephrotic range
MEASUREMENT OF URINARY PROTEIN

Quantitative assessment
 Alternative method - measurement of the total
protein/creatinine ratio (mg/mg) on a spot urine sample .
 best performed on a first morning voided urine specimen to
eliminate the possibility of orthostatic (postural) proteinuria
 normal protein excretion Ratios
 <0.5 in children <2 yr of age
 <0.2 in children ≥2 yr of age.
 A ratio >2 suggests nephrotic-range proteinuria.
Historical Questions

 Is there any Hx of fever, strenuous exercise, or stress?


 Is there a recent Hx of URI?
 Have you noticed changes with the p.t weight?
 Does the p.t have high BP?
 Is there a family Hx of any of kidney disease?
 Is the urine red?
 Does the p.t have any chronic medical conditions? DM?
SLE?
 Are there any associated complaints such as abdominal Pain
or joint pan or any rashes?
Physical examination

1) Skin exam :

 Facial / periorbital edema


 Non blanching pupuric rash
 Malar facial rash
 Lower limb edema (pitting edema)
 Skin rash
2) Abdominal exam :
 Ascites
 Abdominal pain
3) Musculoskletal exam :
 Joint pain or swelling
4) Genital exam :
 Scrotal edema suggests nephrotic range
proteinuria
Management

 A child with persistent proteinuria should


initially receive a physical examination, blood
pressure measurement, urinalysis, and blood
tests for creatinine and urea nitrogen levels
every six to 12 months.

 Treatment of the underlying cause is a must .


Management

 Patients with idiopathic nephrotic syndrome should


receive a trial of prednisone (2 mg per kg per day).

 If steroid therapy fails or adverse effects are


intolerable, second-line therapy (e.g.,
cyclophosphamide, cyclosporine [Sandimmune]) 

  In patients with renal dysfunction, an ACE inhibitor can


be used to decrease proteinuria and slow progression
of renal disease.
Nephrotic syndrome

 Nephrotic syndrome (NS) is


characterized by :
1. persistent heavy proteinuria
(mainly albuminuria) (>2
g/m2/24 h);
2. hypoproteinemia (serum
albumin 250 mg/dL);
3. edema
4. hyper lipidemia.
Clinical signs
 Clinical signs of the nephrotic syndrome are:
 • Periorbital oedema (particularly on waking), the earliest
sign
 • Scrotal or vulval, leg and ankle oedema
 • Ascites
 • Breathlessness due to pleural effusions and abdominal
distension.
Etiology
 The cause of the condition is unknown, but a few cases are
secondary to systemic diseases such as Henoch–Schönlein
purpura (HSP) and other vasculitides, e.g. systemic lupus
erythematosus (SLE), infections (e.g. malaria) or allergens
(e.g. bee sting).
Complication
 Hypovolaemia. During the initial phase of oedema formation the
intravascular compartment may become volume depleted. The child
who becomes hypovolaemic characteristically complains of abdominal
pain and may feel faint. There is peripheral vasoconstriction and
urinary sodium retention.

 Thrombosis. A hypercoagulable state, due to urinary losses of


antithrombin, thrombocytosis which may be exacerbated by steroid
therapy, increased synthesis of clotting factors and increased blood
viscosity from the raised haematocrit, predisposes to thrombosis. This
may affect the brain, limbs and splanchnic circulation with potentially
catastrophic results.

 Infection. Children in relapse are at risk of infection with capsulated


bacteria, especially Pneumococcus. Spontaneous peritonitis may
Approach to
Hematuria and UTI
References for this topic : USMLE Step 2 CK l Pediatrics + Illustrated Textbook of
Paediatrics + Nelson Essentials of Pediatrics 7th Edition
Objectives :
 Define hematuria.
 Discuss the methods of detection of hematuria.
 Recognize the general classification of hematuria.
 Discuss the commonest causes of hematuria in pediatrics.
 Discuss the key components from the history and physical examination in evaluating
a child with hematuria.
 Identify the distinguishing features between glomerular and non-glomerular causes of
hematuria based on urine analysis.
 Formulate an appropriate management plan to evaluate an infant with hematuria.
 Define UTI.
 List the risk factors
 List the clinical presentation.
 Recognize how to investigate radiologically.
 Outline the radiological investigations required.
 Discuss the management.
Hematuria , the general classification , the commonest
causes in pediatrics.

 hematuria : the presence of red blood cells (erythrocytes) in the


urine.
 Blood in the urine can come from the kidney or anywhere in the
urinary tract .
 Classification :
 Gross hematuria –means that blood can be seen in the urine with
the naked eye because it turns the urine pink, red, or tea-colored.
 Microscopic hematuria, defined as more than 3 to 5 red blood
cells (RBCs) per high-power field on freshly voided and centrifuged
urine, is often benign.
  The most common identifiable lower urinary tract causes of
hematuria include urinary tract infection (UTI), kidney stones, and
hypercalciuria.
Discuss the key components from the history and
physical examination in evaluating a child with
hematuria.

 History :
 A history of passage of clots in urine? In Extraglomerular cause
 Hematuria due to glomerular causes is painless history .
 A history of fever, abdominal pain, dysuria, frequency, and recent enuresis ? UTI
 A history of recent trauma to the abdomen ? In Hydronephrosis
 A history of a recent throat or skin infection ? In postinfectious glomerulonephritis
 A history of joint pains, skin rashes ? In HSP and SLE
 Physical :
 skin examination ? Purpura
 abdominal examination ? palpable kidneys
 Genitalia
 ophthalmological evaluation
Discuss the methods of detection of
hematuria
 history and urine examination
 Urinalysis : Dip strip analysis or dipstick test : dip
the strip in the urine, tap off excess urine, and read
the strip at the recommended time (usually 1 min). A
freshly voided urine should be used.
 Urine culture: A midstream or clean-catch specimen
of urine : whenever a urinary tract infection is
suspected.
 BUN/serum creatinine: Elevated levels suggest
significant renal disease as the cause of hematuria.
 CBC counts and, platelet counts : bleeding disorder
 Urine calcium: calcium-creatinine ratio can be
helpful in establishing hypercalciuria as a cause of
hematuria.
 Phase-contrast microscopy : for the presence of a
significant number of dysmorphic RBCs
features between
glomerular and non-
glomerular causes of
hematuria based on
urine analysis
Formulate an appropriate management plan to
evaluate an infant with hematuria.

 ? ?? ? ? !!
Additional : DISEASES PRESENTING PRIMARILY WITH
HEMATURIA

 Acute Poststreptococcal Glomerulonephritis :


 − Follows infection with nephrogenic strains of group A beta-hemolytic
streptococci of the throat or skin .
 − 1–2 weeks after strep pharyngitis or 3–6 weeks after skin infection
(impetigo)
 –(classic triad) : Edema, hypertension, hematuria
 • Diagnosis :
 − Need positive throat culture or increasing antibody titer to streptococcal
antigens; best single test is the anti-DNase antigen
 - RBC casts
 − Low C3 (returns to normal in 6–8 weeks)
 IgA Nephropathy (Berger disease) :
 Most common chronic glomerular disease worldwide
 Clinical presentation :
 – Most commonly presents with gross hematuria in association with upper
respiratory infection or gastrointestinal infection
 – Then mild proteinuria, mild to moderate hypertension
 – Normal C3
 Most important primary treatment is blood pressure control.

 Alport Syndrome :
 glomerulonephritis, end-stage kidney disease, and hearing loss.
 Hereditary nephritis (X-linked dominant)
 1–2 days after upper respiratory infection
 Asymptomatic hematuria
 Hearing deficits
 Ocular abnormalities
 renal biopsy shows foam cells
 Hemolytic Uremic Syndrome (HUS) :
 Most common cause of acute renal failure in young children
 • Prognosis—more than 90% survive acute stage; small number develop ESRD (end-stage renal
disease)
 − Most common <4 years old
 Most from E. coli O157:H7 (shiga toxin–producing) : from undercooked meat or unpasteurized
milk . Also from Shigella, Salmonella, Campylobacter.
  characterized by triad of : micro-angiopathic hemolytic anemia, thrombocytopenia,  and renal
injury ( uremia )
 • Pathophysiology : mesangial deposits of granular, amorphous material—vascular occlusion,
glomerular sclerosis, cortical necrosis → localized clotting → damage to RBCs as they pass
through vessels → Intrarenal platelet adhesion and damage
 (abnormal RBCs and platelets then removed by liver and spleen)
 helmet cells, burr cells, fragmented cells, platelets usually 20,000–100,000/mm3, low-
grade microscopic hematuria and proteinuria
 • Clinical presentation :
 − Bloody diarrhea
 − 5–10 days after infection, sudden pallor, irritability, weakness, oliguria occur; mild
 renal insufficiency to acute renal failure (ARF)
 • Treatment : fluids and electrolytes , dialysis , Plasmapheresis if no diarrhea or CNS problem .
 − No antibiotics if E. coli O157:H7 is suspected—treatment increases risk of
developing HUS
URINARY TRACT INFECTION (UTI)
 UTI more common in boys than in girls until after second year
 • Etiology—colonic bacteria (mostly E. coli, then Klebsiella and Proteus;
some S. saprophyticus)
 Types :

Bladder Cystitis Kidneys Pyelonephritis Asymptomatic

• positive urine culture 
• dysuria, urgency,  • abdominal or flank pain, 
without signs or 
frequency, suprapubic  fever, malaise, nausea, 
symptoms; can
pain, incontinence, no  vomiting, diarrhea;
• become symptomatic if 
fever (unless very  • nonspecific in newborns 
untreated; almost 
young) and infants
exclusive to girls
 UTI in childhood is important because:
• up to half of patients have a structural abnormality of their urinary tract
• pyelonephritis may damage the growing kidney by forming a scar,
predisposing to hypertension and to chronic renal failure if the scarring is
bilateral.

 Vesicoureteric reflux important risk factor :


 is a developmental anomaly of the vesico-ureteric junctions.
 abnormal backflow of urine from bladder to kidney
 multiple urinary tract infections since birth
 − Predisposition to pyelonephritis → scarring → reflux nephropathy
(hypertension, proteinuria, renal insufficiency to end-stage renal
disease [ESRD], impaired kidney growth)
 • Diagnosis – VCUG for diagnosis and grading
 • Grading
 – Grade I: into nondilated ureter (common for anyone)
 – Grade II: upper collecting system without dilatation
 – Grade III: into dilated collecting system with calyceal blunting
 – Grade IV: grossly dilated ureter and ballooning of calyces
 – Grade V: massive; significant dilatation and tortuosity of ureter;
intrarenal reflux with blunting of renal pedicles
 Risk factors  • Treatment
 − Females: ° Wiping °  − Lower-urinary tract infection (cystitis) with
Sexual activity ° Pregnancy amoxicillin, trimethoprim-sulfamethoxazole,
 − Males— uncircumcised  or nitrofurantoin (if no fever)
 − Both:  − Pyelonephritis start with oral antibiotics,
unless patient requires hospitalization and
 ° Vesicoureteral reflux
 IV fluids
 ° Toilet-training
 ° Constipation
 ° Anatomic abnormalities  • Follow up
 − Do urine culture 1 week after stopping
 • Diagnosis antibiotics to confirm sterility; periodic
 —urine culture (gold standard)— reassessment
and UA findings  for next 1–2 years
 − Need a proper sample—if  − Obtain ultrasound for anatomy, suspected
toilet-trained, midstream
abscess, hydronephrosis, recurrent UTI
collection; otherwise, suprapubic
tap or catheterization  − Obtain voiding cystourethrogram (VCUG) in
 − Positive if >50,000 colonies/mL
recurrent UTIs or UTIs with complications
(single pathogen) plus pyuria  or abnormal ultrasound findings
Recognize how to investigate
radiologically

• An ultrasound scan is usually the first


procedure that is used to assess the urinary
tract.

• A dimercaptosuccinic acid (DMSA) scan is a


type of scan that is used to assess the state of
the kidney. DMSA is a mildly radioactive
substance that shows up on a special camera,
known as a gamma camera.

• Micturating cystourethrogram (MCUG) is a


similar procedure to a DMSA scan, except that it
is used to study the bladder rather than the
kidneys. heck that child does not have
vesicoureteral reflux .
Child with chronic
. diarrhea

References for this topic : USMLE Step 2 CK l Pediatrics + Illustrated Textbook of


Paediatrics + nelson
Define chronic diarrhea.
 diarrhea is excessive daily stool liquid volume (>10 mL stool/kg body
weight/day).
 chronic diarrhea loose or watery stool lasts or excessively frequent stools,
or stools that are large in volume for more than 2 weeks .
 has a wide range of possible causes, including serious and benign conditions
that are more difficult to diagnose.
Differentiate between acute and chronic
diarrhea
 Acute diarrhea the duration is less than 14 days
 Most acute diarrhea is viral and is self-limited,
requiring no diagnostic testing or specific intervention .
 Bacterial agents tend to cause much more severe
illness and typically are seen in food-associated outbreaks
or in regions with poor public sanitation .
 Bacterial enteritis should be suspected when there is
dysentery (bloody diarrhea with fever) and whenever
severe symptoms are present .
Mechanism of chronic diarrhoea.

 Secretory diarrhea occurs when the intestinal mucosa


directly secretes fluid and electrolytes into the stool and is
the result of inflammation (e.g., inflammatory bowel disease,
chemical stimulus).
 Secretion also is stimulated by mediators of inflammation and by
various hormones, such as vasoactive intestinal peptide
secreted by a neuroendocrine tumor.
 Cholera is a secretory diarrhea stimulated by the enterotoxin
of Vibrio cholerae, which causes increased levels of cAMP within
enterocytes and leads to secretion into the small-bowel lumen.
 Osmotic diarrhea occurs after malabsorption of an
ingested substance, which “pulls” water into the bowel
lumen.
 A classic example is the diarrhea of lactose
intolerance.
 Osmotic diarrhea also can result from generalized
maldigestion, such as that seen with pancreatic
insufficiency or with intestinal injury.
 Certain nonabsorbable laxatives : polyethylene
glycol and milk of magnesia
 Fermentation of malabsorbedsubstances (e.g.,
lactose) often occurs, resulting in gas, cramps, and
acidic stools.
 A common cause of chronic loose stools in early childhood is
functional diarrhea, commonly known as toddler’s
diarrhea.
 This condition is defined by frequent watery stools in the
setting of normal growth and weight gain and is caused by
excessive intake of sweetened liquids .
Causes of Malabsorption
 cystic fibrosis is most common congenital disorder associated with
malabsorption
 celiac diseases.
 Food allergy and intolerance
 Infection : Giardiasis common primary infection causing chronic
malabsorption
 protein-losing enteropathy
 disaccharide deficiency
 Most common anomaly causing incomplete bowel obstruction with
malabsorption is malrotation .
 Celiac disease—associated with exposure to gluten (rye, wheat, barley,
derivatives)
 Others .
Describe how to approach child with chronic diarrhea

 history should include :


 the onset of diarrhea .
 number and character of stools .
 estimates of stool volume .
 presence of other symptoms, such as blood in the stool, fever, and weight
loss.
 Recent travel and exposures should be documented.
 dietary factors should be investigated .
 a list of medications recently used should be obtained.
 Factors that seem to worsen or improve the diarrhea should be
determined.
 Physical examination should be thorough :
 evaluating for abdominal distention .
 tenderness .
 quality of bowel sounds .
 presence of blood in the stool or a large fecal mass on
rectal examination .
 anal sphincter tone.
 Laboratory testing should include :
 stool culture .
 Complete blood count if bacterial enteritis is suspected.
 If diarrhea occurs after a course of antibiotics, a Clostridium
difficile toxin assay should be ordered .
 if stools are reported to be oily or fatty, fecal fat content or fecal
elastase to test for pancreatic insufficiency should be measured.
 Tests for specific diagnoses should be sent when appropriate
(e.g., serum antibody tests for celiac disease; colonoscopy for
suspected ulcerative colitis).
 A trial of lactose restriction for several days or lactose breath
hydrogen testing is helpful in the evaluation of lactose intolerance.
 Steatorrhea is most prominent with pancreatic insufficiency ( cystic
fibrosis ) all require a sweat chloride
Innumerate some effect of chronic diarrhea.

 dehydration
cystic fibrosis
 PANCREATIC DISEASE
 CF is a chronic progressive disease that can present with protein and fat
malabsorption
 The cause of inadequate pancreatic digestive function in 95% of cases is
cystic fibrosis
 CFTR mutations : The gene for CF, located on the long arm of chromosome
7, encodes for a polypeptide, the cystic fibrosis transmembrane
regulator (CFTR),
 autosomal recessive disorder .
 The secretory and absorptive characteristics of epithelial cells are affected
by abnormal CFTR, resulting in the clinical manifestations of CF.
 The altered chloride ion conductance in the sweat gland results in
excessively high sweat sodium and chloride levels. This is the basis of the
sweat chloride test,
 elevated sweat chloride > 60 mEq/L in 99% of patients with CF .
 (failure to thrive, hypoalbuminemia, steatorrhea),
 liver disease (cholestatic jaundice),
 chronic respiratory infection
 Ninety percent of patients with CF are born with exocrine pancreatic
insufficiency This leads to malabsorption of proteins, sugars (to a
lesser extent), and especially fat.
 Fat malabsorption manifests clinically as steatorrhea (large foul-
smelling stools), deficiencies of fat-soluble vitamins (A, D, E, and K),
and failure to thrive.
 Protein malabsorption can present early in infancy as hypoproteinemia
and peripheral edema .
celiac diseases

 Celiac disease is an injury to the mucosa of the small


intestine caused by the ingestion of gluten (a protein
component) from wheat, rye, barley, and related grains .
 Patients mostly age 6 months to 2 years
 Permanent intolerance .
 Rice does not contain gluten and can be eaten freely .
 celiac disease causes malabsorption and malnutrition.
 The disease is seen in association with type 1 diabetes,
thyroiditis, Turner syndrome, and trisomy 21.
 Symptoms : Diarrhea, abdominal bloating, failure to
thrive, irritability, decreased appetite, Pubertal Delay and
ascites caused by hypoproteinemia are classic.
 Blood for anti-tissue transglutaminase (IgA) , serum IgA
(false if IgA deficiency) (best initial test)
 Definitive test—small intestine biopsy is essential to
confirm the diagnosis and should be performed while the
patient is still taking gluten.
 Treatment—lifelong, strict gluten-free diet
Discuss the investigation of chronic
diarrhoea

 Laboratory testing should include :


 stool culture .
 Complete blood count if bacterial enteritis is suspected.
 If diarrhea occurs after a course of antibiotics, a Clostridium
difficile toxin assay should be ordered .
 if stools are reported to be oily or fatty, fecal fat content or fecal
elastase to test for pancreatic insufficiency should be measured.
 Tests for specific diagnoses should be sent when appropriate (e.g.,
serum antibody tests for celiac disease; colonoscopy for suspected
ulcerative colitis).
 A trial of lactose restriction for several days or lactose breath hydrogen
testing is helpful in the evaluation of lactose intolerance.
 Steatorrhea is most prominent with pancreatic insufficiency ( cystic
fibrosis ) all require a sweat chloride
 Common causes of bloody diarrhea:
Additional  Campylobacter
 Amoeba (E. histolytica)
 Shigella
 E. Coli
 Salmonella
Child with Developmental
Delay & Cerebral Palsy

References for this topic : nelson+ Illustrated Textbook of Paediatrics


Learning Objectives:
 1. Define Cerebral Palsy.
 2. Epidemiology of CP
 3. Recognize the risk factors.
 4. Classify morphological types of C.P.
 5. Recognize associated disabilities.
 6. Recognize the difference between C.P. and neurodegenerative disorder.
 7. List of differential diagnosis
 8. Discuss the investigation
 9. Discuss the management.
 10. Define mental retardation.
 11. Classify the severity of mental retardation.
 12. Present a brief list of the differential diagnosis of developmental delay.
 13. Recognize the early signs of mental retardation and cerebral palsy.
Definition
 Cerebral palsy (CP) refers to a group of nonprogressive, but often
changing, motor impairment syndromes secondary to anomalies
or lesions of the brain arising before or after birth.

Epidemiology
 The prevalence of CP at age 8 in the United States is 3.6 per 1000;
prevalence is much higher in premature and twin births.
 Prematurity and low birth weight infants (leading to perinatal asphyxia),
congenital malformations, and kernicterus are causes of CP noted at
birth.
 Ten percent of children with CP have acquired CP, developing at later
ages. Meningitis and head injury (accidental and nonaccidental) are the
most common causes of acquired CP (Table 10-10).
 Nearly 50% of children with CP have no identifiable risk factors. As
genomic medicine advances, many of these causes of idiopathic CP may
Risk factors
 Most children with CP, except in its mildest forms, are
diagnosed in the first 18 months of life when they fail to
attain motor milestones or show abnormalities such as
asymmetric gross motor function, hypertonia, or
hypotonia.
 CP can be characterized further by the affected parts of
the body (Table 10-11) and descriptions of the
predominant type of motor disorder (Table 10-12).
 Comorbidities in these children often include epilepsy,
learning difficulties, behavioral challenges, and
sensory impairments.
 Many of these children have an isolated motor defect.
Some affected children may be intellectually gifted.
Neurodegenerative disorders and C.P
 Neurodegenerative disorders encompass a large
heterogeneous group of diseases that result from
specific genetic and biochemical defects and varied
unknown causes. Neurodegenerative disorders can
present at any age.
 neurologic deterioration may be demonstrated as loss
of speech, vision, hearing, and intellectual or motor
abilities—sometimes in concert with seizures, feeding
difficulties, and mental retardation.
 Progression may be slow over many years, or may lead
to death in early childhood.
Differential Diagnosis
 - Cerebral palsy.
 - Congenital myopathy.
 - Spinal cord lesion
 - Global developmental delay as in many syndromes
or unidentified cause.
 - Metabolic Neuropathy
 - Traumatic Peripheral Nerve Lesions
 - Subdural hematoma
 - stroke
Imaging

 Ultrasonography: can delineate clear-cut structural abnormalities


and show evidence of hemorrhage or hypoxic-ischemic injury.

 MRI: it is most useful after 2-3 weeks of life and is the diagnostic
neuroimaging study of choice for older children. Also for
determination of appropriate myelination for a given age.

 CT brain: helps to identify congenital malformations, intracranial


hemorrhage, and periventricular leukomalacia more clearly than
ultrasonography.
 Treatment depends on the pattern of dysfunction.

 Physical and occupational therapy can facilitate optimal positioning


and movement patterns, increasing function of the affected parts.
 Spasticity management also may include oral medications
(dantrolene, benzodiazepines, and baclofen), botulinum toxin
injections, and implantation of intrathecal baclofen pumps.
 Management of seizures, spasticity, orthopedic impairments, and
sensory impairments may help improve educational attainment.
 CP cannot be cured, but a host of interventions can improve
functional abilities, participation in society, and quality of life.
 Like all children, an assessment and reinforcement of strengths are
important, especially for intellectually intact or gifted children who
have simple motor deficits.
Medical Management
 The goal of pharmacotherapy in patients with cerebral
palsy is to reduce symptoms and prevent
complications.
 Neuromuscular Blockers.
 Botulinum Toxins: lower limb spasticity
 Muscle relaxants.
 Antiparkinsonian, anticonvulsant,
antidopaminergic, and antidepressant agents
Surgical treatment

 Orthopedic procedures are usually a last resort for


children with severe spasticity and/or fixed
contractures/deformities:
 Repair of scoliosis and hip dislocation. Most common
conditions requiring surgery.
 Tendon lengthening or transfer to decrease the
imbalance from muscle spasticity.
 Osteotomy to realign a limb.
Mental Retardation

 Also known as intellectual disability (ID).


 Defined as significantly subnormal intellectual
functioning for a child’s developmental stage, existing
concurrently with deficits in adaptive behaviors (self-
care, home living, communication, and social
interactions).
 The etiology of the central nervous system insult
resulting in MR may involve genetic disorders,
teratogenic influences, perinatal insults, acquired
childhood disease, and environmental and social
factors.
Mental Retardation

 Also known as intellectual disability (ID).


 Defined as significantly subnormal
intellectual functioning for a child’s
developmental stage, existing
concurrently with deficits in adaptive
behaviors (self-care, home living,
communication, and social interactions).
 The etiology of the central nervous
system insult resulting in MR may involve
genetic disorders, teratogenic influences,
perinatal insults, acquired childhood
disease, and environmental and social
factors.
Differential diagnosis of
developmental delay
 Autism
 Borderline intellectual functioning
 Childhood Disintegrative Disorder
 Cerebral palsy
 Cognitive Deficits
 Learning Disorder, Reading, Written Expression,
Mathematics
 Pediatric Depression
 Pervasive Developmental Disorder
 Pediatric attention deficit hyperactivity disorder
 Rett Syndrome
 Severe communication/language disorders
Early signs of mental retardation
 Language delay: including speech and understanding.
 Red flags include:
1. no mama/dada/babbling by 12 months,
2. no 2-word phrases by age 2,
3. parents reporting concern that the child may be deaf.
 Fine motor/adaptive delay: such as self-feeding, toileting, and
dressing.

 Cognitive delay: difficulties with memory, problem-solving and logical


reasoning, difficulty following directions.

 Social delays: lack of interest in age-appropriate toys and delays in


imaginative play and reciprocal play with age-matched peers.
 Gross motor Delays: Subtle delays in gross motor acquisition,
or clumsiness, may be identified in the developmental
assessment.
 Behavioral disturbances: more likely to have difficult
temperaments, hyperactivity, disordered sleep, defiance,
inattention.
 Neurologic and physical abnormalities: Prevalence of ID is
increased among children with seizure disorders, microcephaly,
macrocephaly, history of intrauterine or postnatal growth
retardation, prematurity, and congenital anomalies.
Child with Rickets

References for this topic : USMLE Step 2 CK l Pediatrics + Illustrated Textbook of


Paediatrics + Nelson Essentials of Pediatrics 7th Edition
Objective :
 Understand calcium and phosphate homeostasis.
 Discuss the common causes of rickets in children.
 Describe the pathophysiology and clinical
presentation of various types of rickets.
 List the laboratory and radiological findings and
explain the management of different types of rickets.
Understand calcium homeostasis
 Calcium is the most abundant cation in the body (1–1.5 kg)
 99% of calcium is present in skeleton , the remaining 1% is in
Extracellular fluid ,intracellular and cell membrane .
 Calcium regulation involves
• three tissues - bone, intestine, kidney
• three hormones - PTH, calcitonin, activated vitamin D
 100 mg/day (10%) of ingested calcium is excreted in urine.
 Ionized and complex form – filtered in glomerulus and 70% of filtered calcium is
reabsorbed in the proximal tubule, 20% in the loop of Henle, and 9% in the distal
tubule.

Functions
 Structural integrity and metabolism of bone (bone growth and remodeling).

 Tooth formation.

 Coenzyme function - e.g. in haemostatic (clotting cascade).

 Synaptic transmission.

 Stimulus – secretion coupling: in nerve terminals; endocrine and exocrine glands (exocytosis).
REGULATION OF
CALCIUM
 Three hormones- PTH, calcitonin,
activated vitamin D
 The regulatory mechanisms
comprise:
• Intestinal absorption :duodenum
and proximal jejunum
• Renal tubular reabsorption and
excretion
• Exchange of calcium between
plasma and bone

 Plasma calcium homeostasis


depends on - balance between the
hypocalcaemic effects of calcitonin
and the hypercalcaemic effects of
PTH, vitamin D and calcium intake
REGULATION OF Phosphate
 Organic phosphate compounds are present in
the structural units of cell, and inorganic
phosphate is associated with calcium in bone
and teeth.
 The total phosphorus in the adult human
body is about 12 g/kg.
 Serum phosphate plays an important role in
regulating neutrality.
  The body needs phosphorus to build and repair
bones and teeth, help nerves function, and
make muscles contract. Most (about 85%) of
the phosphorus contained in phosphate is found
in bones. The rest of it is stored in tissues
throughout the body.
 The blood level is held relatively constant by
regulating phosphate excretion by the kidney
 Studies have noted that disorders of increased
fibroblast growth factor 23 (FGF-23) function are
associated with rickets.
Actions of PTH :
GIT
Uptake calcium
Increased absorption of phosphate
Kidney
Increased reabsorption of calcium
Increased urinary excretion of
phosphate
Actions of Vitamin D3
• Increases intestinal calcium and phosphate absorption
• Increases calcium and phosphate reabsorption in kidneys
• Increases calcium transporters through cellular membranes in bone –
increases bone resorption
Discuss the common causes of rickets in
children

 Vitamin D deficiency
usually presents with
bony deformity and the
classical picture of
rickets.
 Rickets signifies a
failure in mineralisation
of the growing bone.
Failure of mature bone
to mineralise is
osteomalacia.

• The predominant cause nutritional vitamin D deficiency due to


inadequate intake or insufficient exposure to direct sunlight.
• Anticonvulsant drugs (eg, phenobarbital, phenytoin) accelerate
metabolism of calcidiol, which may lead to insufficiency and rickets
Describe the pathophysiology and clinical
presentation of various types of rickets.

 Pathophysiology : In the vitamin D deficiency


state, hypocalcemia develops, which stimulates
excess secretion of parathyroid hormone. In turn,
renal phosphorus loss is enhanced, further
reducing deposition of calcium in the bone.
 Early in the course of rickets, the calcium
concentration in the serum decreases.
 After the parathyroid response, the calcium
concentration usually returns to the reference
range, though phosphorus levels remain low.
 Alkaline phosphatase, which is produced by
overactive osteoblast cells, leaks into the
extracellular fluids, so that its concentration rises
to anywhere from moderate elevation to very
high levels.
types of rickets ?
 Nutritional rickets : results from inadequate sunlight exposure or inadequate intake of dietary vitamin
D, calcium, or phosphorus.
 Vitamin D–dependent rickets (Type 1) : Is secondary to a defect in the gene that codes for the
production of renal 25(OH) D3–1α-hydroxylase.
 Vitamin D–dependent rickets (Type 2) : Is a rare autosomal recessive disorder caused by mutations in
the vitamin D receptor. It does not respond to vitamin D treatment; elevated levels of circulating calcitriol
is an obvious sign for this type.
clinical presentation
 The earliest sign of rickets is a
ping-pong ball sensationof the
skull (craniotabes)

 Craniotabes is softening or
thinning of the skull in infants
and children
 The costo chondral
junctions may be palpable
(rachitic rosary)
 ,wrists (especially in
crawling infants)
 and ankles (especially in
walking infants) may be
widened
 and there may be a
horizontal depression on
the lower chest
corresponding to
attachment of the softened
ribs and with the
diaphragm (Harrison
sulcus) .
 The legs may become
bowed
List the laboratory and radiological findings and
explain the management of different types of rickets.

 • Blood tests – serum calcium is low or normal, phosphorus low, plasma


alkaline phosphatase activity greatly increased, 25-hydroxyvitamin D
may be low and parathyroid hormone elevated.

 The most common laboratory findings in nutritional rickets are:

 Decreases in serum calcium, serum phosphorus, calcidiol, calcitriol,


urinary calcium.

 increase Parathyroid hormone, alkaline phosphatase,urinary


phosphorus levels are elevated.
List the
laboratory
radiological
findings
1- widening of the distal epyphysis .
2- fraying and widening of the
metaphysis.
3- angular deformities of the arm and
leg bone.
4- Cupping in the metaphysis .

 https://
emedicine.medscape.com/article
/412862-overview#a2
management
NUTRITIONAL VIT D DEFICIENCY:
Vitamin D + Calcium + Phosphorus
 2 strategies for vitamin D administration :
1 - Stoss therapy: 300,000-600,000 IU of vitamin D oral or IM as 2-4 doses
over 1 day.
2 - Alternative: Daily, high-dose vitamin D, with doses ranging from 2,000-
5,000 IU/day over 4-6 wk.

 Either strategy should be followed by daily vitamin D intake of 400 IU/day.


 Ensure enough dietary calcium & phosphorus.
 Healing occurs in 2–4 weeks and can be monitored from the lowering of
alkaline phosphatase, increasing vitamin D levels and healing on X-rays, but
complete reversal of bony deformities may take years.
Symptomatic hypocalcaemia:
 IV Calcium acutely, followed by oral calcium supplements, over 2-6 wk in
children who receive enough dietary calcium.
Childhood obesity

References for this topic : USMLE Step 2 CK l Pediatrics + nelson


Define obesity in pediatrics, and differentiate it from
overweight

 Obesity is the most common nutritional disorder affecting children and


adolescents in the developed world. 
 BMI >95% for age and sex is diagnostic of obesity .
 85 to 95% = overweight .
Identify correct BMI percentile
categories for children
 Obesity and overweight are defined using BMI percentiles
  According to the CDC children >2 yr old with, a BMI over the 95th percentile indicates 
obesity 
 BMI between the 85th and 95th percentiles indicates overweight.
Understand the causes and contributory factors in
obesity.

 One important risk factor is maternal obesity during pregnancy


 Children born to obese mothers are three to five times more likely to be
obese in childhood.
 Women who gain much more weight than recommended during
pregnancy have children who have a higher BMI than normal in
adolescence.
 some small for gestational age (SGA) newborns have higher risks for
abnormal postnatal weight gain and diabetes.
 (predisposition, parental obesity )
 The associations
between obesity and
television watching and
excessive dietary
intake support the
important influence of
environment .
 family/patient inactivity,
feeding baby as
response to any crying .

 Diseases represent <5% of cases of childhood obesity.


 Prader-Willi syndrome .
 Down syndrome .

 Medical complications are often related to the degree of obesity and


usually decrease in severity or resolve with weight reduction .

 Obesity is associated with the presence of precursors of coronary heart


disease that are already evident in 12- and 13-year-old children .
Describe obesity risk behaviors and the
potential consequences of obesity in children.

 Complications of Obese infants and children are


at increased risk of becoming obese adults
(the risk is greater with advanced age of onset)
 cardiovascular (hypertension, increased
cholesterol)
 slipped capital femoral epithesis
 sleep apnea,
 hyperinsulinism
 type 2 diabetes
 acanthosis nigrans.
Provide an organized weight- management and obesity -
prevention strategies with the patients and parents.

 achieve healthy eating and activity patterns by :


 exercise
 balanced diet
 no medications
List some societal responsibilities
needed to support obesity in
children
Approach to a
wheezy infants
Objectives

 Review the physiological consideration of the Childs airway that


predispose to wheezes.
 Discuss the key history, risk factors, family history and physical finding
associate with wheezing in infant.
 Develop a rational approach to the differential diagnosis and
management of wheezing.
 Identify the life threatening conditions associated with wheeze.
DEFINITION OF WHEEZING
 A wheeze is a continuous musical sound heard during chest auscultation
that lasts longer than 250 msec
 It is produced by the oscillation of opposing walls of an airway narrowed
almost to the point of closure
 It can be high pitched or low pitched, consist of single or multiple notes,
and occur during inspiration or expiration.

 Wheezes can originate from airways of any size throughout the proximal
conducting airways.
 Wheezing requires sufficient airflow to generate airway oscillation and
produce sound in addition to narrowing or compression of the airway.
 Thus, the absence of wheezing in a patient who presents with acute
asthma may be an ominous finding, suggesting impending respiratory
failure.
PHYSIOLOGICAL CONSIDERATION OF THE
CHILDS AIRWAY THAT PREDISPOSE TO
WHEEZING
 The child’s respiratory system, including airways, continues
to mature until at least eight years of age, therefore the
pediatric airway is described and managed differently from
the adult’s.
 Structural and mechanical differences predispose infants and
young children to respiratory compromise.
 The newborn’s larynx is just one-third of the diameter of the
adult larynx. Narrow nasal passages, in combination with
being obligatory nose-breathers up to 5–6 months of age,
means that infants may experience respiratory distress if
nasal passages become oedematous or contain secretions
such as mucus or blood.
 With the airway of an infant measuring around 6 mm in
The pediatric airway is characterized and differentiated
from an adult airway by the following features:
 short maxilla and mandible
 large tongue
 floppy epiglottis
 shorter trachea
 more acute angle of airway, particularly notable
when attempting to visualize with a laryngoscope
 a more cephalad larynx that moves distally as the
neck grows
 the cricoid ring is the narrowest portion of the
airway
Cont.
 smaller lower airways, less developed with fewer
alveoli
 true alveoli not present until 2 months, with full
complement developed by around eight years of age
 little smooth muscle present in airways
 little collateral ventilation in airways.
CLINICAL HISTORY
 Two important aspects of the medical history include the patient's
age at the onset of wheezing and the course of onset (acute versus
gradual)
 Acute onset of wheezing raises the possibility of foreign body
aspiration, particularly if there is a history of choking.
 In addition, it is helpful to distinguish between intermittent and
persistent wheezing.
 Persistent wheezing presenting very early in life suggests a
congenital or structural abnormality.
 In contrast, paroxysmal or intermittent wheezing is a characteristic
finding in patients with asthma.
 Persistent wheezing with sudden onset is consistent with foreign
body aspiration, whereas the slowly progressive onset of wheezing
may be a sign of extraluminal bronchial compression by a growing
mass or lymph node.
Cont.
 Cough is a symptom commonly associated with wheezing
 The nature of the associated cough (wet versus dry) may be
helpful in determining the underlying etiology.
 Wet cough typically results from excessive mucus
production, mostly due to infection or inflammation (eg,
bronchiectasis, cystic fibrosis, asthma, and chronic
aspiration).
 In contrast, pure bronchoconstriction or structural causes for
airway narrowing (eg, asthma, or compression, foreign body,
vascular ring) are usually associated with a dry cough.
 However, the underlying etiology of a dry cough can be
complicated by a secondary process, making this distinction
difficult (eg, mechanical obstruction can lead to impaired
mucus clearance resulting in infection and a wet cough).
Features in the history that favor the
diagnosis of asthma include:
 Intermittent episodes of wheezing that usually are the
result of a common trigger (ie, upper respiratory
infections, weather changes, exercise, or allergens)
 Seasonal variation
 Family history of asthma and/or atopy
 Good response to asthma medications
 Positive asthma predictive index
features that suggest a diagnosis other
than asthma include the following
History
 Onset of symptoms in early infancy
 Neonatal respiratory distress +/- ventilatory support
 Neonatal neurologic dysfunction
 Intractable wheezing unresponsive to bronchodilators
 Wheezing associated with feeding or vomiting
 Difficulty swallowing +/- recurrent vomiting
 Diarrhea
 Poor weight gain
 Stridor
 Oxygen requirement >1 week after acute attack
PHYSICAL EXAMINATION

 General examination of a wheezy child should include


measurement of weight and height, vital signs
including oxygen saturation, and digital inspection for
the presence of cyanosis or clubbing.
Chest examination should focus on
the following features:
 Inspection for the presence of respiratory distress,
tachypnea, retractions, or structural abnormalities.
Pertinent findings include an increased
anteroposterior (AP) diameter associated with chronic
hyperinflation, pectus excavatum caused by chronic
airway obstruction and exaggerated swings in
intrathoracic pressure, or scoliosis complicated by
airway compression.

 Palpation to detect supratracheal lymphadenopathy or


tracheal deviation
Cont.
 Percussion can define the position of the diaphragm and detect differences in
resonance among lung regions, and is the most underperformed part of the
examination.

 Auscultation allows identification of the characteristics and location of wheezing,


as well as variations in air entry among different lung regions.
 A prolonged expiratory phase suggests airway narrowing. Wheezing caused by a
large or central airway obstruction (eg, vascular ring, subglottic stenosis,
tracheomalacia) has a constant acoustic character throughout the lung, but
varies in loudness depending upon the distance from the site of obstruction.
 In contrast, the degree of narrowing varies from place to place within the lung in
the setting of small airway obstruction (eg, asthma, cystic fibrosis, primary ciliary
dyskinesia, aspiration).
 The presence of focal wheezing is usually indicative of a localized and mostly
structural airway abnormality, and therefore airway evaluation by imaging or
bronchoscopy is warranted.
Cont.
 Crackles can be present in conjunction with wheezing
in asthma and in a variety of other conditions, such as
those leading to bronchiectasis (eg, cystic fibrosis,
primary ciliary dyskinesia, immune deficiency).
 Early inspiratory crackles are often present in patients
with asthma due to air flowing through secretions or
slightly closed airways during inspiration.
 Late inspiratory crackles are usually associated with
interstitial lung disease and early congestive heart
failure.
 Thus, the presence of crackles does not exclude the
diagnosis of asthma
Cont.
 Decreased wheezing after bronchodilator therapy is suggestive
of asthma, but does not rule comorbid conditions if clinically
suspected.

The remainder of the examination should focus on cardiac findings,


including murmurs and signs of heart failure.
Examination of the skin for eczema or other cutaneous lesions may
assist in diagnosis. Nasal examination may reveal signs of allergic
rhinitis, sinusitis, or nasal polyps.
DDX

Acute Wheezing
Chronic or Recurrent Wheezing
Management

 Treatment depends on the underling etiology, however the first and


foremost priority is establishing airway and breathing and maintaining
oxygenation.

 Presence of severe respiratory distress warrants hospitalization


Status
Asthmaticus
 Status asthmaticus is considered a
medical emergency.
 It is the extreme form of an
asthma exacerbation that can
result in
hypoxemia,hypercarbia,and
secondary respiratory failure.
Allergies in
Pediatrics

References for this topic : USMLE Step 2 CK l Pediatrics + Illustrated Textbook of


Paediatrics + Nelson Essentials of Pediatrics 7th Edition
Objectives :
 Recognize common skin manifestations of
allergy, and describe the management goals.
(Atopic dermatitis, urticaria, drug rash).
 Recognizes the common symptoms of
allergic-rhinitis, and discuss possible etiology,
 initiate appropriate laboratory studies, and
propose a treatment plan.
 Explore the management of an anaphylactic
episode, and make recommendations for
further precautions.
Recognize common skin manifestations of allergy, and describe the
management goals. (Atopic dermatitis, urticaria, drug rash).
 Eczema is classified as atopic (where there is evidence of IgE antibodies to
common allergens) or non-atopic.

 Atopic Dermatitis (Eczema) : is a type of inflammation of


the skin (dermatitis) It results in itchy, red, swollen, and cracked skin.
 A genetic deficiency of skin barrier function is important in the
pathogenesis of atopic eczema.
 uncommon in the first 2 months but usually in the first year of life
chronic or relapsing - Majority develop allergic rhinitis and/or
asthma
 There is often a family history of atopic disorders: eczema, asthma,
allergic rhinitis (hay fever).
 • Clinical presentation :
 – Intense cutaneous reactivity and pruritus (itch) ; worse at night;
scratching induces lesions; becomes excoriated No itch? – then it’s not
eczema
 – Exacerbations with foods, inhalants, bacterial infection, decreased
humidity, excessive sweating, irritants
 diagnosis is made clinically but they have elevated total plasma IgE
level
 – Patterns for skin reactions:
Atopic Dermatitis (Eczema) :

° Acute: ° Subacute—erythematous,  ° Chronic—lichenification 


 Erythematous papules excoriated, scaling papules (thickening, darkening)
 Intensely pruritic
 Serous exudate, excoriation Subacute and Chronic Atopic Dermatitis Most
Commonly
Affects the Flexural Surfaces of Joints.

 – Distribution pattern:
Atopic Dermatitis (Eczema) :

° Infancy: face, scalp, extensor surfaces of extremities

° Older, long­standing disease: flexural aspects

• Complications

– Recurrent viral skin infections—Kaposi varicelliform 
eruption (eczema herpeticum) most common
Management
 Cutaneous hydration
 ° Dry skin, especially in winter (xerosis) , Avoiding soap, frequently using
emollients? , Avoiding nylon and wool clothes?
 Is there a need to give or change medications:
 Topical corticosteroids
 Seven classes—the higher potency classes are not to be used on face or
intertriginous
 areas and only for short periods
 ° Goal—emollients and low-potency steroids for maintenance

 Immunomodulators Inhibits activation of key cells


 tacrolimus (calcineurin inhibitor)
 Can use as young as 2 years of age
 Safe on face
 Antibiotics or antiviral agents
 Antihistamines (sedating at night; for pruritus )
 Urticaria : and angioedema
 Hives, also known as urticaria, is a kind of skin rash with red, raised, itchy
bumps.
 Papular urticaria is a delayed hypersensitivity reaction most commonly seen
on the legs, following a bite from a flea, bedbug, or animal or bird mite.
 Neonatal urticaria (erythema toxicum) : a common rash appearing at 2–3
days of age

 1- Acute urticaria IgE-mediated (duration <6 weeks) :


 usually results from exposure to an allergen or a viral infection , produce
swelling of the lips and soft tissues around the eyes (angioedema), and even
anaphylaxis.
 IgE-mediated food allergy - Immediate cow’s milk allergy : widespread
urticaria immediately after the first formula feed and lip swelling after .

 2- Chronic urticaria Non IgE-mediated, (persisting >6 weeks):


 is usually non-allergic in origin.
 From – Radiocontrast agents – Viral agents (especially EBV, hepatitis B) –
Opiates, NSAIDs
Radiocontrast agents are substances used to enhance the visibility of internal structures
 3- Physical urticarias :
 induced by environmental stimulis —temperature, pressure, stroking,
vibration, light , Cold, delayed pressure .
 4- Hereditary angioedema
 recurrent episodes of severe swelling - nonpitting edema
 Autosomal dominant
 C1 esterase-inhibitor deficiency

 • Treatment
 – Most respond to avoidance of trigger and oral antihistamine
 – Severe—epinephrine, short-burst corticosteroids
 – If H1 antagonist alone does not work, H1 plus H2 antagonists are
effective; consider
 steroids
 – For chronic refractory angioedema/urticaria → IVIg or plasmapheresis
drug rash
Chapter 85 u Adverse Reactions to Drugs  297

 Drug allergies do occur in children, especially to antibiotics


  Gell and Coombs classification can be used to describe some drug-
induced allergic reactions
allergic-rhinitis - hay fever -‫ىاالاقش‬
‫حم‬
 Allergic rhinitis, also known as hay fever , an allergy caused by pollen
or dust in which the mucous membranes of the eyes and nose are itchy
and inflamed, causing a runny nose and watery eyes.
 6 years of age
 Risk if : early introduction of formula (versus breast milk) or solids, mother
smoking before child is 1 year old, heavy exposure to indoor allergens .
 increased symptoms with greater exposure
 Diagnosis suggested by typical symptoms in absence of URI or structural
abnormality (nasal congestion/pruritus, worse at night with snoring, mouth-
breathing; watery, itchy eyes; postnasal drip with cough; possible wheezing;
headache)
 Physical examination
 Conjunctival injection, chemosis (edema),
 Dennie lines —prominent symmetric skin folds
 Transverse nasal crease (from allergic salute)
 turbinate hypertrophy
initiate appropriate laboratory studies, and propose
a treatment plan.
 Laboratory evaluation (no initial routine labs; clinical DX)
 – In vivo—skin test (best):
 ° Use appropriate allergens for geographic area plus indoor allergens.
 ° May not be positive before two seasons

 – In vitro:
 ° Peripheral eosinophilia
 ° Eosinophils in nasal and bronchial secretions; more sensitive than blood
 eosinophils
 Treatment  : 
 —environmental control plus
removal of allergen is most
effective method
 - house dust mite, cat, cockroach
 – No smoking
 – No wood-burning
stoves/fireplaces
 • Pharmacologic control
 – Antihistamines (first-line
therapy):
 – Intranasal corticosteroids —
most effective medication, but
not first-line:
management of an anaphylactic episode

 Anaphylaxis is a serious allergic reaction that is rapid in onset and may cause death.
 Anaphylaxis Sudden release of active mediators with cutaneous, respiratory,
cardiovascular, gastrointestinal symptoms
 • Presentation —reactions from ingested allergens are delayed (minutes to 2 hours);
with injected allergen, reaction is immediate (more gastrointestinal symptoms)
 • Most common reasons
 – In hospital—latex, antibiotics, IVIg (intravenous immunoglobulin), radiocontrast agents
 – Out of hospital—food (most common is peanuts), insect sting, oral medications,
idiopathic
 most responsive to treatment in its early phases
• Treatment
– What the patient should do immediately:
° Injectable epinephrine
° Oral liquid diphenhydramine
° Transport to ER
– Medical:
° Oxygen and airway management

° Epinephrine IM (IV for severe hypotension); 
intravenous fluid expansion; H1 antagonist; 
corticosteroids; nebulized, short­acting beta­2 agonist 
(with respiratory symptoms); H2 antagonist (if oral 
allergen)
Approach to child with Hemolytic
Anemia (Pallor and Jaundice)

References for this topic : USMLE Step 2 CK l Pediatrics ( mostly ) + nelson +


some tables from illustrated
Recognize the method for detection of pallor

 Pallor is the paleness of skin and mucous membranes, due to the reduced
amount of oxyhemoglobin or decreased peripheral perfusion.
 Sites to look for pallor :
Lower palpebral conjunctiva
Tip and dorsum of the tongue
Soft palate
Nail beds
Palmar or plantar creases
General body skin
Define the causes of pallor and the definition of anemia.

 Causes of pallor :
1 - Anemia 
2 - Pallor without anemia : Physiologic (“fair skinned”) , Shock ,
Hypoglycemia and other metabolic derangements , Respiratory distress
, Skin edema , Pheochromocytoma .

 definition of anemia : either hypoproduction or increases distraction .


 Hemoglobin concentration or RBC mass less than 5th percentile for age
and gender .
Classify hemolytic anemia.

 The most common RBC membrane


disorders : are hereditary spherocytosis
and hereditary elliptocytosis .
 RBC enzyme deficiencies may lead to
hemolysis : glucose-6-phosphate
dehydrogenase (G6PD) deficiency and
pyruvate kinase deficiency.
 Congenital hemolytic disorders
(enzyme deficiencies and membrane
problems) often present in the first 6
months of life and frequently are
associated with neonatal jaundice
 Hemoglobinopathy :
Hemoglobin SS, S-C,
S- thalassemia
Discuss the general approach to hemolysis
including history, examination and investigation.

 A history of jaundice, pallor, previously affected siblings,


 drug ingestion by the mother, or excessive blood loss at the time of
birth provides important clues to the diagnosis in newborns.
 in patients with hemolytic anemias are jaundice, pallor, and
splenomegaly. Because of increased bilirubin production, gallstones
(bilirubinate),
 a result of chronic hemolysis, are a common complication. Systemic
complaints suggest acute or chronic illnesses as probable causes of
anemia
 Congenital hemolytic disorders (enzyme deficiencies and
membrane problems) often present in the first 6 months of life and
frequently are associated with neonatal jaundice,
 acute blood loss and acute hemolysis, manifesting as
tachycardia, blood pressure changes, and, most ominously,
an altered state of consciousness.
 The presence of jaundice suggests hemolysis.
 Hepatosplenomegaly and adenopathy suggest infiltrative
disorders.
Describe the clinical presentation of sickle cell
anemia
 Newborn usually without symptoms
 development of hemolytic anemia over first 2–4 months
(replacement of HbF)
 as early as age 6 months some children have functional asplenia; by
age 5 all have functional asplenia .
 First presentation usually hand-foot syndrome (acute distal
dactylitis)—symmetric,
painful swelling of hands and feet (ischemic necrosis of small bones) .
 Acute painful crises: Younger—mostly extremities , With increasing
age—head, chest, back, abdomen Precipitated by illness, fever,
hypoxia, acidosis, or without any factors (older)
 vaso-occlusive crises → ischemic damage .
 Acute splenic sequestration .
 Altered splenic function → increased susceptibility to
infection, especially with encapsulated bacteria (S.
pneumococcus, H. influenzae, N. meningitidis) .
 Aplastic crisis—after infection with parvovirus B19;
absence of reticulocytes during
 acute anemia

 Aplastic crisis—after infection with parvovirus


B19; absence of reticulocytes during
 acute anemia
 Note : sickle cell anemia always associated with
pain .
Discuss the management of sickle cell anemia.
 Aim : Prevent complications
 Immunize (pneumococcal regular plus 23-valent, meningococcal) .
 Start penicillin prophylaxis at 2 months until age 5 .
 Educate family (assessing illness, palpating spleen, etc.)
 Folate supplementation
 Aggressive antibiotic treatment of infections
 Pain control
 Transfusions as needed
 −Monitor for risk of stroke with transcranial Doppler
 Hydroxyurea
 Bone-marrow transplant in selected patients age <16 years
Discuss the complication of sickle cell anemia.
Discuss the long-term health maintenance of sickle cell
disease.

Vasoocclusive crises should be minimised by avoiding exposure to cold,


dehydration, excessive exercise, undue stress or hypoxia.

 Because of increased susceptibility to infection:


Immunization against pneumococcal, HiB and meningococcus infection.
 Lifelong folic acid supplementation: 1 mg\day
Describe the thalassemia syndromes
 Alpha thalassemia trait: deletion of 2 genes : Mild hypochromic,
microcytic anemia (normal RDW) without clinical problems .
 HgB H disease: deletion of 3 genes; Hgb Barts >25% in newborn period
and easily diagnosed with electrophoresis .
At least one parent has alpha-thalassemia trait; later beta-tetramers develop
(Hgb H—interact with RBC membrane to produce Heinz bodies) and can be
identified; electrophoretically microcytosis and hypochromia with mild to
moderate
Anemiat arget cells present, mild splenomegaly, jaundice and cholelithiasis
Typically do not require transfusions or splenectomy .
 Alpha-thalassemia major: deletion of 4 genes severe
fetal anemia resulting in hydrops fetalis
Newborn has predominantly Hgb Barts with small amounts of
other fetal Hgb .
immediate exchange transfusions are required for any
possibility of survival
transfusion-dependent with only chance of cure (bone
marrow transplant)
Describe the clinical presentation of
thalassemia.

 Excess alpha globin chains → alpha tetramers


form; increase in HbF (no problemwith gamma-
chain production)
 Presents in second month of life with progressive
anemia, hypersplenism, and cardiac
decompensation (Hb <4 mg/dL)

 Expanded medullary space with increased


expansion of face and skull
(hair-on end);
 extramedullary hematopoiesis
 hepatosplenomegaly
Discuss the complications of
thalasemia.
Discuss the management of
thalassemia.
 − Transfusions
 − Deferoxamine (assess iron overload with liver biopsy)
 − May need splenectomy
 − Bone-marrow transplant curative
Discuss G6PD deficiency anemia.

 X-linked
 Within 24–48 hours after ingestion of an oxidant
(acetylsalicylic acid, sulfa drugs,
antimalarials, fava beans) or infection and
severe illness → rapid drop in Hb,
hemoglobinuria and jaundice (if severe) .
 Acute drop in Hb, saturated haptoglobin → free
Hb and hemoglobinuria, Heinz bodies,
increased reticulocytes
 Diagnosis—direct measurement of G6PD
activity .
 Treatment—prevention (avoid oxidants);
supportive for anemia .
Approach to a child
with abdominal pain

References for this topic : nelson+ Illustrated Textbook of Paediatrics


Objectives
 Describe Approach Abdominal Pain by history, examination and proper investigation.
 Outline Differential diagnosis between:
  
 Acute Abdominal pain
  
 Define acute abdomen.
 Discuss the most common etiologies of acute abdomen.
 Narrow the differential diagnosis of acute abdominal based on the location of the pain and age of the patient.
 Mention the appropriate diagnostic and imaging studies in evaluation of a patient with acute abdomen.
 Identify the red flag symptoms in patients with acute abdomen that indicate urgent surgical consult.

  
 Chronic and Recurrent abdominal pain.
  
 Identify organic abdominal pain.
 Define constipation & list its differential diagnosis according to the age:
  
 Neonatal
 Infancy
 Childhood
PRESENTATION

o Acute pain o Chronic pain


o Intense
pain with o  Periods of remissions
dramatic onset & exacerbations with
intervals of relief in
o Reachmaximum in between,
hours or days
o Months or years
Lab work

CBC
LFT
Amylase and Lipase
Urinalysis
Pregnancy test
Stool hemoccult
Plain radiographs:

 demonstrate signs of
obstruction ( distended
bowel) or perforation (such
as free air)
 
 Fluid-filled loops of small
bowel can be seen with
gastroenteritis.

 For children who may have


midgut volvulus, an upper
GI contrast series is the best
examination to visualize the
duodenum.
Cont’

Ultrasonography:
 Gallstones.

 Genitourinary conditions
(eg, ovarian torsion,
ruptured ovarian cyst, and
testicular torsion).

 Intussusception.

 Appendicitis.
 Computed tomography (CT) with contrast is useful for the
evaluation of patients with acute abdominal pain when a
wide variety of diagnoses are being considered ,such as :
(appendicitis, pancreatitis, intraabdominal abscess, blunt
abdominal trauma, and for the evaluation of an
intraabdominal mass).
Acute abdominal pain
Definition:
Acute abdomen represents:
 Rapid onset of severe symptoms.
 May indicate potentially life-threatening intra-
abdominal pathology .
 Requires urgent surgical intervention, not all acute
abdominal pain needs emergency intervention.
 In nearly half of the children admitted to hospital, the
pain resolves undiagnosed.
Acute appendicitis

 Inflammation of appendix caused by occlusion of its


lumen commonly by fecalith
 It’s the most common cause of abdominal pain require
surgical treatment rare in infancy (rare below 3 years
old) because:wide base-short in length but can affect
any age
 cause: most common organism is E.coli –sterptococal
faecalis  which spread from lumen of intestine rarly
from blood
 history(symptoms):
 1. acute colicky abdomial pain started in para umbilical area then
shifted to right iliac fossa (from localised peritoneal infammation -same
supply by T10)
 2. vomiting+anorexia
 3. fever may present +-
 examination(signs):
 1.Abdominal pain aggravated by movement Ex:
walking,coughing,jumping,bumps on the road
 2.localized tenderness (McBurney’s point)
 3.Rebound tenderness at same point= McBurney’s sign
 4.Rovsings sign=palpation of left iliac fossa pain in right iliac fossa
(due shifting of gases)
 investigation:
 1.CBC: high WBC (neutrophilis) (normal WBC does not
exlude daignosis)
 2.urine analysis: high pus cell
 3.abdominal US:show thickened, with increased blood
flow
 4.laproscopy: diagnostic and theraputic. In preschool
children
 The diagnosis is more diffcult and Perforation may be
rapid and the signs are easy to underestimate at this age
 treatment: if not comlicated: appenectomy (open or
laproscopic)
Intussusception
 Describes the invagination of
proximal bowel into distal segment
Leading to intestinal obstruction.
 Sex: male>female (3:1)
 Age: 3 mths 2yrs (more than 50%
of cases below 12 mts)
 It’s the most common cause of
intestinal obstruction in infancy
beyond neonatal period.
 site: 1.ileocaecal (ileocolic):
(commomn site) ileum passing into
the caecum through the ileocaecal
 2.ileoileal
 3.others : as jejunojejunal,
jejunoileal, ileoileal
 Types:

 1. idiopathic intussusception: starts at the


ileocolic junction and affects infants and
toddlers

 2. enteroenteral intussusception:
jejunojejunal, jejunoileal, ileoileal which
occurs in older children may be secondary
to other problems.
 Cause: Unknown
 but there many Risk factors as:
 1. Lymphoid hyperplasia
 2. Meckels diverticulum
 3. Henoch-Schönlein purpura [HSP]
 4 cystic fibrosis
 5. ascaris lumbricoides
 6. juvenile inflammatory polyp
 clinical pictures:
 1. sudden sever crampy paroxysmal abdominal pain usualy relifed after
vomiting or pass stool
 2. vomiting
 3. pallor
 4. shocked or dehydrated
 5. abdominal distention 6. sausage like mass in right upper abdomen
 7. PR=passage of red current jelly stool (late sign present in 60% of
cases)

 Radiology :
 1.xray abdomin:distended small bowel with abscent Of gases in distal
colon and rectum
 2.US abdomin: helpful both to confirm The diagnosis and to check
response to treatment
 3.barium enema: diagnostic and therabutics Show:
 1. claw-sign 2. Coiled-spring
 treatment: Correct water and electrolyte imbalance
 NGT for decompression
 Barium or air enema: 70% success rate\increase % of recurrence
 Surgery done in: 1.faliure of reduction by enema 2.signs and symptoms
suggest peritonitis
Chronic and Recurrent abdominal
pain
 Defined as pain sufficient to interrupt normal
activities and lasts for at least 3months.
 10% of school age children

• Organic disorders include problems that have a specific cause. Organic


disorders generally cause pain much less often than functional disorders
(problems without a known cause).
Constipation
 definition: >two or fewer stools per week or passage of
hard,pellet-like stools for at least 2 weeks. (compare it with
normal happit).
 (causes of Constipation) DDX:
  functional: most common cause
  neuromuscular :Hirschsprung Disease (important cause
specially in infancy)
  metabolic :Hypercalcemia-Hypokalemia
  endocrine: Hypothyroidism  structural: Imperforate Anus-
intestinal obstruction
  Neurofibromatosis
  medication: opiate-anticholinergic  others: cystic fibrosis-
Clostridium botulinum infection in infance .
 Causes according to age:
  In young infants>> a dietary transition (eg, from breast
milk to formula, the addition of solid foods into the diet,
from formula to whole milk).
  In toddlers>> toilet training. In toddlers or following an
illness associated with either a severe diaper dermatitis
or dehydration.
  In older children>> time of school entry, because they
refuse to defecate while they are at school.
  other causes >> inattention to defecate –anal fissure
Approach to
unconjugated
Hyperbilirubinemia

References for this topic : nelson


Define hyper-billirubenimeia

 Hyperbilirubinemia happens
when there is too much bilirubin
in newborn blood .
  is a life threatening disorder in
newborns .
 the physiological jaundice is the
most prevalent type however in
some regions pathological
jaundice is also common.
Recall the physiological steps in the metabolism of
bilirubin.

 Bilirubin is produced by the catabolism of hemoglobin in the


reticuloendothelial system.
 The tetrapyrrole ring of heme is cleaved by heme oxygenase to form
equivalent quantities of biliverdin and carbon monoxide.
 Because no other biologic source of carbon monoxide exists, the excretion
of this gas is stoichiometrically identical to the production of bilirubin.
 Biliverdin is converted to bilirubin by biliverdin reductase.
 One gram of hemoglobin produces 35 mg of bilirubin.
 Sources of bilirubin other than circulating hemoglobin
represent 20% of bilirubin production
 these sources include inefficient shunt) hemoglobin
production and lysis of precursor cells in bone marrow.
 newborns have a twofold to threefold greater rate of bilirubin
production than adults .
 unconjugated bilirubin is toxic to the central nervous system
and is insoluble in water limiting its excretion .
 Unconjugated bilirubin binds to albumin on specific bilirubin
binding sites .
 If the binding sites become saturated or if a competitive
compound binds at the site, displacing bound bilirubin,
 free bilirubin becomes available to enter the central nervous
system.
 Organic acids such as free fatty acids and drugs such as
sulfisoxazole can displace bilirubin from its binding site on
albumin .
 The enzyme glucuronosyltransferase represents the rate-
limiting step of bilirubin conjugation.
List the key component from the history and physical
examination in patient with jaundice.

 https://emedicine.medscape.com/article/974786-clinical
 Read from the link

 o Did it start suddenly or gradually? o Abdominal pain?


(Onset: acute or chronic) o Nausea?
 o How did you first notice it and where? o Vomiting?
(Eye or skin) o Dark urine?
 o Is it getting worse? (Progression) o Pale stool?
 o Are there any precipitating factors? o Itching?
(Precipitating factor) o History of contact with hepatitis
 o Is there anything that makes it worse? patient?
(Aggravating factor) o History of exposure to Radiation?
 o Is there anything that makes it better? (Cancer risk)
(Relieving factor)
 o Is the discoloration continuous or
intermittent?
 o If intermittent, ask about frequency.
Differentiate between physiological and pathological
causes of jaundice.

 Physiologic jaundice is a common cause of


hyperbilirubinemia among newborns.
 It is a diagnosis of exclusion
 Physiologic jaundice is the result of many factors that are
normal physiologic characteristics of newborns such as :
 Increased bilirubin production resulting from an increased
RBC mass .
 shortened RBC life span .
 hepatic immaturity of ligandin and glucuronosyltransferase .
 Physiologic jaundice may be exaggerated among infants of
Greek and Asian ancestry.
 The clinical pattern of physiologic jaundice in term infants includes a
peak indirect-reacting bilirubin level of no more than 12 mg/dL on day 3
of life.
 In premature infants, the peak is higher (15 mg/dL) and occurs later
(fifth day).
 The peak level of indirect bilirubin during physiologic jaundice may be
higher in breast milk–fed infants than in formula-fed infants (15 to 17
mg/dL versus 12 mg/dL).
 Jaundice is unphysiologic or pathologic if it is clinically evident on
the first day of life,
 if the bilirubin level increases more than 0.5 mg/dL/hr,
 if the peak bilirubin is greater than 13 mg/dL in term infants,
 if the direct bilirubin fraction is greater than1.5 mg/dL,
 or if hepatosplenomegaly and anemia are present.
 Crigler-Najjar syndrome is a serious, rare, autosomal recessive,
permanent deficiency of glucuronosyltransferase that results in severe
indirect hyperbilirubinemia .
Crigler-Najjar syndrome
 rare inherited disorder affecting the metabolism of bilirubin
 jaundice is apparent at birth or in infancy. Severe unconjugated
hyperbilirubinemia can lead to a condition called kernicterus, which is
a form of brain damage caused by the accumulation of unconjugated
bilirubin in the brain and nerve tissues.
 Type II responds to enzyme induction by phenobarbital, producingan
increase in enzyme activity and a reduction of bilirubin levels.
 Type I does not respond to phenobarbital and manifests as persistent
indirect hyperbilirubinemia, often leading to kernicterus.
Gilbert disease
 caused by a mutation of the promoter
region of glucuronosyltransferase and
results in a mild indirect
hyperbilirubinemia.
 liver does not properly process bilirubin.
 may become apparent shortly after
birth but generally diagnosed after
puberty
 In the presence of another icterogenic
factor (hemolysis), more severe
jaundice may develop .
Breast milk jaundice

 Breast milk jaundice may be associated with unconjugated


hyperbilirubinemia without evidence of hemolysis
 during the first to second week of life. Bilirubin levels rarely increase to more
than 20 mg/dL.
 Interruption of breastfeeding for 1 to 2 days results in a rapid decline of
bilirubin levels,
 which do not increase significantly after breastfeeding resumes.
 Breast milk may contain an inhibitor of bilirubin conjugation or may increase
enterohepatic recirculation of bilirubin because of breast milk
glucuronidase.
 Jaundice on the first day of life is always pathologic, and
immediate attention is needed to establish the cause. Early
onset often is a result of hemolysis, internal hemorrhage
 (cephalhematoma, hepatic or splenic hematoma), or infection
 Infection also is often associated with direct-reacting bilirubin
resulting from perinatal congenital infections or from bacterial
sepsis.
 Physical evidence of jaundice is observed in infants when
bilirubin levels reach 5 to 10 mg/dL (versus 2 to 3 mg/dL in
adults).
Discusses the differential diagnosis of indirect
hyperbilirubinemia.
Elaborate on the different treatment modalities in jaundice
and list the common indications and side effects of each.

 Phototherapy is an effective and safe method for reducing


indirect bilirubin levels, particularly when initiated before
serum bilirubin increases to levels associated with
kernicterus.
 In term infants, phototherapy is begun when indirect
bilirubin levels are between 16 and 18 mg/dL.
 Phototherapy is initiated in premature infants when bilirubin
is at lower levels, to prevent bilirubin from reaching the
high concentrations .
 Complications of phototherapy include an
increased insensible water loss, diarrhea, and
dehydration.
 Additional problem are macular-papular red
skin rash, lethargy, masking of cyanosis, nasal
obstruction by eye pads, and potential for
retinal damage.
 Skin bronzing may be noted in infants with
direct-reacting hyperbilirubinemia
 . Infants with mild hemolytic disease of the
newborn occasionally may be managed
successfully with phototherapy for
hyperbilirubinemia, but care must be taken to
follow these infants for the late occurrence of
anemia from continued hemolysis.
 Exchange transfusion usually is reserved for infants with
dangerously high indirect bilirubin levels who are at risk for
kernicterus.
 As a rule of thumb, a level of 20 mg/dL for indirect- reacting bilirubin
is the exchange number for infants with hemolysis who weigh more
than 2000 g.
 Asymptomatic infants with physiologic or breast milk jaundice may
not require exchange transfusion, unless the indirect bilirubin level
exceeds 25 mg/dL.
 the level in an infant weighing 1500 g would be 15 mg/dL.
 Infants weighing less than 1000 g usually do not require an exchange
transfusion until the bilirubin level exceeds 10 mg/dL.
 Complications of exchange transfusion include problems related to the blood
(transfusion reaction, metabolic instability, or infection),
 the catheter (vessel perforation or hemorrhage), or the procedure (hypotension or
necrotizing enterocolitis [NEC]).
 Unusual complications include thrombocytopenia and graft-versus-host disease.
 Continuation of phototherapy may reduce the necessity for subsequent exchange
transfusions.
Describe the most fatal complications of hyper-
billrubinemia.

 Prolonged hyperbilirubinemia (severe jaundice) can result in chronic


bilirubin encephalopathy (kernicterus)
 Infants with kernicterus may have a fever or seizures. High pitched
crying is an effect of kernicterus. Scientists used a computer to record
and measure cranial nerves 8, 9 and 12 in 50 infants who were divided
into two groups equally depending upon bilirubin concentrations. Of
the 50 infants, 43 had tracings of high pitched crying
 https://en.wikipedia.org/wiki/Neonatal_jaundice
Approach to suspected
child Abuse
References for this topic : nelson+ Illustrated Textbook of Paediatrics
Objectives :
 Define Child abuse and Non Accidental Injury NAI.
 Differentiate between types of Child abuse.
 List the features in the history that should alert the
physician to suspect child abuse.
 List the features in clinical examination that should
raise the suspicion of child abuse and NAI.
 Discuss the approach to discussing NAI with the
family.
 Describe, briefly, the legal repercussion of diagnosis
of NAI.
Define Child abuse and Non Accidental Injury NAI.

 – Child maltreatment— abusive actions or acts of commission and


lack of action, or acts of omission that result in morbidity or death
 – Physical abuse— intentional injuries to a child by a caregiver that
result in bruises, burns, fractures, lacerations, punctures, or organ
damage; also may be accompanied by short- or long-term emotional
consequences
 – Psychological maltreatment— intentional verbal or behavioral
acts or omissions that result in adverse emotional consequences—
spurning, exploiting/corrupting, withholding emotional
responsiveness, isolating, terrorizing
 – Sexual abuse— any act intended for sexual gratification of an
adult
 – Factitious disorder— intentionally giving poisons or toxins, or any
other deceptive action to simulate a disorder
Differentiate between types of Child abuse.

 Read kaplan chapter Child Abuse and Neglect 7 .


Differentiate between types of Child abuse.
 child abuse is categorised into:
 1- • Physical abuse : involve hitting, shaking, throwing, poisoning, burning or
scalding, drowning, suffocating
• When to suspect ?
– Injury is unexplained or implausible
– Injury is incompatible with the history given or with child’s level of development
– There are no reports of death or serious brain injury from witnessed falls <10 feet.
Clinical Findings :
 Bruises • Most common
How to know it’s no accidental ?
1. bruises in various stages are not compatible with a single event .
2. Accidental it will be thin, leading surfaces overlying bone edges (e.g., shins)
3. Non- accidental it will be in buttocks, genitals, back, back of hands, thoraco-abdominal
 Result in battered child syndrome : complex of physical injuries (as fractures,
hematomas, and contusions) that results from gross abuse
 Battered child syndrome is suggested by bruises, scars, internal organ damage, and
fractures in various stages of healing.
Approach to
conjugated
hyperbilirubinemia

References for this topic : nelson+ Illustrated Textbook of Paediatrics


Objectives

 List the key component from the history and physical examination in
patient with jaundice.
 Discusses the differential diagnosis of direct hyperbilirubinemia.
 Define and enumerate the laboratory findings in cholestatic jaundice.
 List the commonest causes of cholestatic jaundice in infancy and
childhood.
 Discuss the pathogenesis, clinical presentation and outline the
management of billiary atresia.
 Conjugated hyperbilirubinaemia (>25
µmol/L) is suggested by the baby
passing dark urine and unpigmented
pale stools.
 Hepatomegaly and poor weight gain
are other clinical signs that may be
present.
 Its causes include neonatal hepatitis
syndrome and biliary atresia, with
improved prognosis of biliary atresia
with early diagnosis.
Biliary atresia : also known as extrahepatic ducto-penia , one
or more bile ducts are abnormally narrow, blocked, or absent.

• This occurs in 1 in 14000 live births. It is a progressive disease, in which there


is destruction or absence of the extrahepatic biliary tree and intrahepatic
biliary ducts.
• The jaundice of extrahepatic biliary atresia (biliary atresia) usually is
not evident immediately at birth, but develops in the first week or two
of life.

• The reason is that extrahepatic bile ducts are usually present at birth,
but are then destroyed by an idiopathic inflammatory process.

• Aside from jaundice, these infants do not initially appear ill.

• The liver injury progresses rapidly to cirrhosis; symptoms of portal


hypertension with splenomegaly, ascites, muscle wasting, and poor
weight gain are evident by a few months of age.

• If surgical drainage is not performed successfully early in the course


(ideally by 2 months), progression to liver failure is inevitable.
Types
Pathogenesis
• Treatment consists of surgical bypass of the fibrotic ducts,
hepatoportoenterostomy (Kasai procedure), in which a loop of
jejunum is anastomosed to the cut surface of the porta hepatis,
facilitating drainage of bile from any remaining patent ductules.

• If surgery is performed before the age of 60 days, 80% of children


achieve bile drainage. The success rate diminishes with increasing
age – hence the need for early diagnosis and treatment.
Child with recurrent infection
(immunodeficiency & HIV
Infection)
: Objectives
 Describe the types of Immunodeficiency
 Identify the clinical features that characterize primary
immunodeficiency
 Identify the basic diagnostic tests that identifies
primary immunodeficiency
 Outline the strategies used to prevent and treat
infection in children with primary immunodeficiency
 Describe the methods of transmission of HIV
 Identify the diagnostic tests to diagnose neonatal and
pediatric HIV
 Outline the treatment of pediatric HIV
Describe the types of Immunodeficiency

 Immune deficiency may be:


1. • Primary (uncommon) – an intrinsic defect in the immune system ,
Many are inherited as X-linked or autosomal recessive disorders.

2. • Secondary (more common) – caused by another disease or


treatment, such as an intercurrent bacterial or viral infection,
malignancy, malnutrition, HIV infection, immunosuppressive therapy,
splenectomy or nephrotic syndrome.
Identify the clinical features , basic diagnostic tests , that characterize primary immunodeficiency

Fr0m :
Illustrated
Textbook of
Paediatrics

From USMLE Step


:
Clinical Presentation : Recurrent Opportunistic infections
with chronic diarrhea and failure to thrive
Initial Labs : Definitive :
• Absolute lymphocyte count • Mitogen stimulation for function
• Normal precludes Dx • Then flow cytometry
• If low → Candida intradermal skin test • Then molecular Dx if no previous
• Normal precludes all FH

Definitive Tests to Make the Specific Diagnosis : T-cell deficiency :


1. • Enumerate T cells (as above for CD2, CD3, CD4, CD8)
2. • Needs to be performed on any infant who is lympho­penic because of possibility 
of SCID 
3. • T­cell function :  – Mitogen stimulation (PHA concanavalin A, pokeweed 
mitogen); measurement of cytokine production
Identify the clinical features , basic diagnostic tests , that characterize primary immunodeficiency

Clinical Presentation : Recurrent infections with


encapsulated bacterial, enteroviral and hepatitis viruses

Initial Labs : Definitive :


• Quantitative • Flow cytometry using monoclonal
immunoglobulins antibodies to B cell-specific CD antigen
• Normal IgA excludes most • Then, molecular Dx if no previous FH

Definitive Tests to Make the Specific Diagnosis : Agamma-globulinemia :


1. • Enumerate blood B cells—flow cytometry using dye-conjugated
monoclonal antibodies to B-cell–specific CD antigens (CD19, CD20)
Identify the clinical features , basic diagnostic tests , that characterize primary immunodeficiency

Phagocytic Clinical Presentation :


Recurrent staph and gram negative
infections
Initial Labs : Definitive :
• Respiratory burst assay using rhodamine dye • Molecular DX
(replaced NBT)
• Flow cytometry for CD antigens specific for
leukocyte adhesion deficiencies

Definitive Tests to Make the Specific Diagnosis : Phagocytic cell defects :


1. • Neutrophil respiratory burst; best way is rhodamine dye (has replaced
the NBT test [nitroblue tetrazolium test])
Identify the clinical features , basic diagnostic tests , that characterize primary immunodeficiency

Clinical Presentation :
1. • Pyogenic infections
2. • Life-threatening septicemia
3. • Neisseria infections

Initial Labs : Definitive :


• CH50 1. • Specific complement assays
2. • Molecular DX
Initial Screening
for Recurrent
Infections
Defects of Antibody Defects of Cellular Immunity Phagocytic Defects :
Production (T cell Defects) : • More severe; no
(B-cell Defects) : survival beyond infancy or early childhood Chronic granulomatous
(without definitive treatment)
disease (CGD)
Bruton DiGeorge syndrome :
agammaglobulinemia : • X-linked and autosomal
• chromosome 22 microdeletion recessive
• X-linked : boys • Dysmorpho-genesis of the third
• Xq22 (Bruton tyrosine kinase) and fourth pharyngeal • Celles can ingest but not kill
pouches; microdeletion 22q11.2. catalase-positive
• Clinical :
• Clinical :
microorganisms S. aureus
• Fine until age 6–9 months
– Thymic hypoplasia (most common )
• Extracellular pyogenic – Dysmorphic features
infections (S. pneumoniae, H. 1. Mandibular hypoplasia • Clinical :
influenza, M. pneumoniae); 2. Hypertelorism (widely spaced • Recurrent lymphadenitis,
hepatitis viruses, enteroviruses eyes) pneumonia, skin infections
(CNS infections) 3. Fish mouth • Osteomyelitis at multiple
sites
• lymphoid hypoplasia (tonsils – Parathyroid hypoplasia causes
• Hepatic abscesses
and adenoids; no splenomegaly neonatal hypocalcemic seizures;
or lymphadenopathy) tetany
– Anomalies of great vessels and – Treatment—bone marrow
• Treatment— monthly IVIg Other CHD (ASD, VSD) transplant
(intravenous immune globulin) and
antibiotics for infections • Treatment—transplantation of
Outline the strategies used to prevent and treat
infection in children with primary
immunodeficiency
Case#1 (Bruton,s disease)
Case#3 (CGD)
A9 months old boy presented to OPD with Fever • A 2 years old boy was presented to ED because of
cough of 10 days duration. His weight was 4kg and large gluteal abscess. Mother gave h/O recurrent
had pus discharge from both ears. Mother stated pustular rash and recurrent chest infections. He had
that he is never will and many antibiotic courses hepatosplenomegaly. He had a cousin died at age
of 5years because of severe chest infection.
were given for the last 2 months.
His WBC was 40.000/His total IgG is 12g/dl
What is your D.D? What most likely diagnosis?
What other points in the history may help you How to confirm?
What investigations will help you? What are the short and long term plans?
What is your treatment plan now and for the
future?

case #2 (DieGeorge) Case# 5 (HIV)


A 3months old infant present with sever oral thrush
for 2weeks and massive diarrhoea.
Mother gave history of neonatal convulsion. A 7month Chadian girl present to ED with severe
He was found to be FTT and systolic murmur was bronchopneumonia. She was started on ceftriaxone and
heard at the pulmonary area. despite intravenous antibiotics she continued to
What is the likely diagnosis? deteriorate and her chest us getting worse. Her initial
What investigation would be helpful? WBC was 3000/CC with low lymphocyte. Her platelets
What is the short and long term treatment plans was only 59000 Her IgG was 22g/dl. Her new CXR
after deterioration showed interstitial pattern then she
Acquired Immunodeficiency Syndrome (AIDS)
 (HIV) is a lentivirus (a subgroup of retrovirus)  , infects helper T cells (specifically CD4+),
macrophages, and dendritic cells.
 HIV-1 causes 99% of all human cases. HIV-2, is less, causes 1% to 9% of cases in parts of
Africa and is very rare in the United States.
 • Affects >2 million children worldwide
 The major route of HIV infection is mother-to-child transmission (MTCT) :
 which occurs during pregnancy (intrauterine) , at delivery (intrapartum) or through breast-
feeding (postpartum).
 also be trans-mitted by infected blood products, contaminated needles or through child
sexual abuse, but this is uncommon.
 – Most children acquired at birth from an HIV - positive mother.
 – Breast feeding is an important route of transmission.
 – Pregnant females are routinely screened for HIV infection in prenatal labs, unless the
patient refuses.
 Antenatal antiretroviral drugs, active management of labour and delivery, and avoidance
of breast-feeding can reduce vertical transmission of HIV to <1%.
• Clinical presentation :
 newborns: rapid onset of symptoms and AIDS in first few months of life
 − Initial symptoms may include :
1. º Lymphadenopathy
2. º Hepatosplenomegaly
3. º Failure to thrive
4. º Chronic diarrhea
5. º Interstitial pneumonia
6. º Oral thrush
 recurrent bacterial infections : − Opportunistic infections; most common
is PCP Pneumocystis Jiroveci pneumonia , Mycobacterium , Oral
candidiasis .
• Diagnosis
 Serology may be negative in early disease, so repeat titres after 2–4
weeks are advised. Isolation of the organism is difficult.
1. − HIV-DNA by PCR
2. – Maternal HIV IgG antibodies cross the placenta : Screen will be
positive in all newborns up to age 18 months so need 2 of 3 , ⊕ PCR for
HIV in first month of life.
3. − In any child >18 months of age: test for infection through IgG Ab by
ELISA and then confirm with Western blot to establish the diagnosis.

 Children infected with HIV infection SHOULD HAVE ANNUAL TST


Tuberculin Skin Test (TST) .
Outline the treatment of pediatric HIV
1. – Mother : should be on perinatal triple anti-retro-viral therapy and then IV ZDV at start
of labor until cord is clamped . Zidovudine (ZDV), also known as azido-thymidine (AZT), is an
antiretroviral medication .
2. – Infant : should be started on ZDV (birth) until neonatal disease is excluded.
• at 1 month Also start PCP prophylaxis (TMP-SMZ) until disease excluded
• Then do CBC, platelets, CD4 and CD8 counts

• When to treat ? With symptoms or evidence of immune dysfunction, should be treated


with anti-retroviral therapy, regardless of age or viral load.
 Treatment includes combination antiretroviral therapy and prophylaxis against Pneumocystis
Jiroveci pneumonia (PCP) .
 The drug of choice for early uncomplicated cases over 12 years of age is doxycycline, and
for younger children, amoxicillin.
 Intravenous treatment with ceftriaxone is required for carditis or neurological disease.
  Combination therapy with highly active antiretroviral therapy (HAART)  is recommended
based on the risk of disease progression as determined by CD4 count and plasma HIV RNA
copy number . prevents opportunistic infections that often lead to death.
Approach to lymphadenopathy

References for this topic : nelson + internet


Define lymphadenopathy

 Lymphadenopathy is enlargement of lymph nodes and occurs in response


to a wide variety of infectious, inflammatory and malignant processes.
 Lymphadenitis is acute or chronic inflammation of lymph nodes.
Provide a summary of nodal
distribution and anatomic
drainage.
 The lymphatic system is an open circulatory system that is a component of
the immune system.
 It includes lymph, lymphatic vessels, lymph nodes, spleen, tonsils,
adenoids, Peyer patches, and the thymus.
 Lymph contains lymphocytes and is an ultrafiltrate of blood that is
collected in lymphatic capillaries present throughout the body in all organs
except the brain and heart.

 All lymphatic vessels empty its contents (lymph) into the blood stream.


 Lymph from the upper body enter the venous circulation through two
routes :
1-Lymphatic vessels from the left side of the head, left arm and thoracic
cavity empties lymph into the thoracic duct. This in turn drains into the blood
stream at the junction of the left internal jugular and left subclavian vein.
2-Lymphatic vessels from the right side of the head, right arm and right side
of the thoracic cavity empties lymph into the right lymph duct. This in turn
drains into the blood stream at the junction of the right subclavian vein and
internal jugular vein.
Discuss the differential diagnosis of localized and
generalized lymphadenopathy.

 Generalized lymphadenopathy is enlargement of two or more


noncontiguous lymph node groups (Systemic disease )

 regional lymphadenopathy involves one lymph node group only .


Most commonly cervical then inguinal Can be infection/inflammation in the
area
drained by that node or infection of node itself

 Noninfectious causes of cervical swelling and/or lymphadenopathy


include congenital and acquired cysts, Kawasaki disease, sarcoidosis,
benign neoplasms, and malignancies.
 The differential diagnosis for generalized lymphadenopathy
includes:
 juvenile idiopathic arthritis
 systemic lupus erythematosus
 serum sickness
 adverse drug reactions such as : phenytoin and other antiepileptic
medications
 allopurinol, isoniazid, anti thyroid medications .

 an occasional cause of recurrent fever and cervical lymphadenitis


 A syndrome of periodic fever,
 aphthous
 Stomatitis .
 Pharyngitis .
 adenitis
 Infections :
Bacterial :
Localized: Staph aureus, GAS, cat scratch, tularemia, diphtheria
Generalized : Brucellosis, leptospirosis, typhoid
 Viral : EBV, CMV, HSV, HIV, Hep B, Measles, Mumps, Rubella, Dengue
Fever
 Mycobacterial :TB, Atypical mycobacteria
 Fungal : Coccidiomycosis, Cryptococcosis, Histoplasmosis
 Protozoal : Toxoplasmosis, Leishmaniasis .
Develop a systematic approach to the evaluation
and management of lymphadenopathy.

 Management of lymphadenopathy and lymphadenitis depends on


 patient age
 associated findings
 node size and location,
 severity of the acute systemic symptoms
 In children most cases of cervical lymphadenopathy, without
other signs of acute inflammation, require no specific therapy
and usually regress within 2 to 3 weeks.
 If there is Progression to lymphadenitis or development of
generalized lymphadenopathy requires further evaluation.
 The specific treatment of cervical lymphadenitis depends on the
underlying etiology.
 Empirical treatment targeting S. aureus and group A
streptococcus includes a penicillinase- resistant penicillin (e.g.,
oxacillin) or first-generation cephalosporin (e.g., cefazolin).
 For patients with hypersensitivity to β-lactam antibiotics, or if
community-acquired methicillin-resistant S. aureus ( MRSA ) is
suspected, clindamycin is appropriate.
 Absence of a clinical response within 48 to 72 hours is an
indication for further laboratory evaluation and possible
excisional biopsy and culture.
Recognize worrisome features of
lymphadenopathy that should prompt a referral
for a biopsy
 Systemic symptoms (fever >1 week, night sweats, weight loss [>10% of body weight])
 Supraclavicular (lower cervical) nodes
 Generalized lymphadenopathy
 Fixed nontender nodes in the absence of other symptoms; matted nodes
 Lymph nodes >1 cm with onset in the neonatal period
 Lymph nodes ≥2 cm in diameter that increase in size from baseline or do not respond to two
weeks of antibiotic therapy
 Abnormal chest radiograph (particularly mediastinal mass or hilar adenopathy)
 Abnormal complete blood count (eg, lymphoblasts, cytopenias in more than one cell one)
 Absence of symptoms in the ear, nose, and throat regions
 Persistently elevated ESR/CRP or rising ESR/CRP despite antibiotic therapy
Additional
 EBV is the primary cause of infectious mononucleosis,
 Lymphangitis is an inflammation of subcutaneous lymphatic
channels that presents as an acute bacterial infection, usually
caused by Staphylococcus aureus and group A streptococci.
 Cervical lymphadenitis is the most common regional
lymphadenitis among children and is associated most commonly
with pharyngitis caused by group A streptococcus .
 inguinal regional lymphadenopathy also include sexually
transmitted infections
 submandibular, axillary or inguinal regions may normally be
palpable in healthy people
Approach to a
limping child

References for this topic : nelson+ Illustrated Textbook of Paediatrics


Objectives

 Define limping Child illustration of gates & Classifications of limping.


 Recognize the causes of limping according to the age.
 Recognize the types of limping (painful vs. painless)
 List the important points in the history& physical signs. Clinical
examination of musculoskeletal system. Technique of joints examination
specially hip joints
 List important investigation.
 Discuss the approach to limping child via history examination and
investigation
 Discuss management of medical causes.
Definition

 Normal gait has a stance phase and swing phase; each leg
should have symmetrical timing with each phase.
 The stance phase represents 60% of the gait and begins with
foot contact (usually the heel strike) and ends with the toe-off.
 During the swing phase (40%), the foot is off the ground. The
gait cycle is the interval between stance phases on the same
limb.

 Limp is defined as an uneven, jerky, or laborious gait, usually


caused by pain, weakness, or deformity. It is a type of
asymmetric abnormality of the gait.
Types
 An antalgic gait is a painful limp; the
stance phase and stride of the affected
limb are shortened to decrease the
discomfort of weight bearing on the
affected limb.
 The Trendelenburg gait has a normal
stance phase, but excessive swaying of
the trunk
 Waddling gait refers to a bilateral
decrease in function of the gluteus
muscles.
 Toe walking is a common complaint in
early walkers. A physician should evaluate
any child older than 3 years of age who
still toe walks. Although this is most likely
habit, a neuromuscular disorder (cerebral
palsy, tethered cord), Achilles tendon
contracture (heel cord tightness), or a leg-
length discrepancy should be considered.
Approach
 History
 Examination
 Investigation
 Management

History
 Age and Gender
 Onset and Duration Red flags
 Painful or Painless  Fever
 Trauma  Weight loss
 Associated with swelling, weight loss or fever  Loss of sensation
 Family history  Loss of motor function
 Past medical history
Remarkable history points in wish to identify
possible cause

 Duration : recent : trauma or acute infection /


Chronic: overuse syndromes, apophysitis, Legg-Calve-
Perthes disease, slipped capital femoral epiphysis
(SCFE), or systemic illness (rheumatic disease, tumor)
 Trauma: soft tissue injuries / fractures
 Fever:  osteomyelitis , septic arthritis, Toxic synovitis
with viral illness.
Rheumatic disease & leukemia , rarely.
Pain characteristics :

 Constant , localized, reproducible : fractures, septic arthritis,


osteomyelitis, and sickle cell disease.
 Intermittent moderate pain: juvenile idiopathic arthritis (JIA), Legg-
Calvé-Perthes disease, slipped capital femoral epiphysis, Osgood-
Schlatter disease, and transient synovitis.
 Knee pain : maybe referred pain from hip pathology.
 Nocturnal or wake the child : neoplastic conditions.
 Bilateral pain: myositis, subsequent to viral infection.
 crawl: pain of the foot.
 Lateral thigh pain may indicate lumbar spine abnormality.
 Pain worsens with activity: stress fracture, overuse injury, or
hypermobility syndrome.
improves with activity: rheumatologic.
Joint examination
technique
 Hip rotation : pain on manipulation due
inflammation, infection or trauma
 Internal rotation: prone position
ankles and feet are then rotated away from the
body and notice the rotation range and either
symptomatic or asymptomatic.
 pelvis kept flat for accurate checking of asymmetry.

 ↓ or absent rotation range (lag) : slipped capital


femoral
epiphysis and Legg-Calvé-Perthes disease
Hip rotation cont.

 Symptomatic (painful) rotation : septic arthritis of the hip


,
transient synovitis , and the pain intolerable.
Galeazzi test aka. Allis /Perkins test

 Indicate :
Developmental hip dysplasia (DDH) / leg length discrepancy
Posterior displacement in
DDH .

 leg length discrepancy:


 congenital aplasia or hypoplasia, developmental dysplasia of the hip, clubfoot,
disuse or paralysis, ischemia, Legg-Calvé-Perthes disease, physeal injury or
malunion after trauma, or a tumor.

Lengthening: hyperplasia (eg, in hemihypertrophy syndromes), arteriovenous


fistula, vascular tumors, and fractures (through distraction or stimulation)
 FABERE test :with the "figure of four" maneuver of
 Flexion:hip and knee,  ABduction ,External Rotation

Downward force on bent knee and the opposite hip

 Extension of the ipsilateral sacroiliac joint.

 A discomfort is a positive sign, in the absence of pain with passive motion of the hip
joint.
Barlow’s & Ortolani’s Tests
Congential Dislocation of the
hip

Barlow’s test : Clunk on


Exit

Ortolani’s Test: Clunk on


Entry
Investigations
 Plain radiographs :
nuclear bone scans, ultrasonography, CT, MRI, and positron
emission tomography (PET).

Laboratory Studies, Occasionally necessary and it includes :


CBC
ESR,CRP
Lyme titers; and blood, wound, joint, periosteum,
or bone cultures for infectious conditions such as septic arthritis or
osteomyelitis.

RF, ANA , and HLA-B27 in suspection of rheumatologic disorders.

CK, aldolase, AST , and dystrophin  striated muscle disease


e.g. (Duchenne/Becker muscular dystrophy).
Management

 Depends on the specific diagnosis:


 Bone/Joint infection : aspiration  Abx
 Septic hip : Surgical drainage: ASAP
 Slipped capital femoral epiphysis : operative reduction and pinning
 Oncologic process (eg, leukemia or bone tumor) : admission ,staging workup
and treatment.
 Afebrile, normal radiographs and pain : NSAID (eg, ibuprofen) and follow up
 Toddler’s fracture suspection : splinting of the leg and follow up

Myositis or transient synovitis ,Febrile children without joint effusion and with
normal radiographic and blood studies : followed up
Child with bleeding
disorder
Objectives :

 Describe the physiology of hemostasis.


 Recognize major causes of bleeding in children.
 Discuss non-thrombocytopenic purpura
 Discuss immune Thrombocytopenia purpura.
 Discuss Hemophilia A.
 Discuss Von-Willibrand disease.
 Identify the complications of plasma factors transfusion.(
physiology of
hemostasis

 https://opentextbc.ca/anatomyandp
hysiology/chapter/18-5-hemostasis
/
 526 Section 20 u Hematology
major causes of
bleeding in children

  Platelet counts less than


150,000/mm3  constitute
thrombocytopenia.
 von Willebrand Disease
(vWD) is the Most common
hereditary bleeding
disorder; autosomal
dominant, but more
females affected
 Hemophilia A (VIII) and B
(IX) all X-linked , 85% are A
and 15% B .
Screening Tests for Bleeding
Disorders
Comparison of Hemophilia A, Hemophilia B, and von Willebrand
Disease
PLATELET DISORDERS
mmune or Idiopathic Thrombocytopenic Purpura (ITP)
• commonest cause of thrombocytopenia in childhood.
•  usually follows an acute viral infection
• Most 1–4 years of age but intell 10 years
•  caused by an antibody (IgG or IgM) that binds to the platelet
membrane
• Physical examination is normal if hepatosplenomegaly and
lymphadenopathy should suggest another disease.
• Clinical presentation :
• - 1–4 weeks after a nonspecific viral infection
• - sudden onset of petechiae and purpura with or without
mucous membrane bleeding • Treatment
• - Most resolve within 6 months − Transfusion contraindicated unless life­
• Complication : threatening bleeding (platelet antibodies
• − <1% with intracranial hemorrhage − 10–20% develop will bind to transfused platelets as well)
chronic ITP − No specific treatment if platelets >20,000 and 
•  The diagnosis of ITP usually is based on clinical presentation
and the platelet count and does not often require a bone
no ongoing bleeding
marrow examination. − If very low platelets, ongoing bleeding that is 
• Labs difficult to stop or life­threatening:
− Platelets <20,000/mm3
– Intravenous immunoglobulin for 1–2 days
− Platelet size normal to increased
− Other cell lines normal
° If inadequate response, then prednisone
− Bone marrow— normal to increasedmegakaryocytes – Splenectomy reserved for older child with 
severe disease
Discuss non-thrombocytopenic
purpura
complications of plasma
factors transfusion

 Post-transfusion purpura (PTP) is an


adverse reaction to a blood
transfusion or platelet transfusion
that occurs when the body produces
alloantibodies to the introduced
platelets' antigens.
 These alloantibodies destroy the
patient's platelets leading to
thrombocytopenia, a rapid decline in
platelet count.
Approach to abnormal head
size

References for this topic : nelson+ Illustrated Textbook of Paediatrics + last


slides of growth lecture
Normal head growth
Measurement and monitoring head circumference
 Measuring the head circumference is an important element of the pediatric physical examination,
especially in the first 3 years after birth, when brain growth is maximal.
 Microcephaly refers to a head size that is 2 standard deviations
(approximately 3rd percentile) below the mean, based on age and sex.
 a small head circumference is a reflection of a small brain.
 Macrocephaly is defined as a head circumference greater than 2 standard deviations
(approximately 97th percentile) above the mean, based on age and sex.
CSF physiology and circulation
Etiology of microcephaly and macrocephaly

 Microcephaly is classified as either primary (genetic), which is almost always present at birth,
Familial – when it is present from birth and development is often normal
An autosomal recessive condition – when it is associated with developmental
delay
 secondary (environmental), which may be present at birth or may develop later
from a postnatal insult .
 Myriad syndromes and metabolic disorders are associated with microcephaly
 Brain growth is rapid during the perinatal period, and any insult (infectious,
metabolic, toxic, vascular) is likely to impair brain growth and result in
microcephaly e.g. perinatal hypoxia, hypoglycaemia or meningitis, when it is
often accompanied bycerebral palsy and seizures
 Rarely, a small head is the result of premature closure of one or more skull
sutures, called craniosynostosis.
 Macrocephaly :
 macrocrania (increased skull thickness) Diseases of bone metabolism or
hypertrophy of the bone marrow cause macrocrania ( imp could come as
scenario ) .
 hydrocephalus (enlargement of the ventricles ) .
 megalencephaly (enlargement of the brain) .
 Megalencephaly may be the result of a significant disorder of brain
development or an accumulation of abnormal metabolic substances .
 If there is a rapid increase in head circumference, raised intracranial pressure
should be excluded.
Clinical approach to infant with large and small head
size.
‫‪Thanks‬‬
‫تم بحمد الله ‪ ،‬اسئل الله ان يوفق كل من عمل في هذا العمل ‪ ،‬ويفتح عليه من أبواب الخير ‪ ،‬دعواتكم لهم‬

Potrebbero piacerti anche